Secrets – Pediatric: Nephrology

Secrets – Pediatric: Nephrology

ACID-BASE, FLUIDS, AND ELECTROLYTES
1. How is the cause of hyponatremia established?
The serum sodium concentration, even in states of volume depletion, reflects the extracellular water or volume status. In children presenting with hyponatremia, the VOLUME status must always be evaluated. The causes of hyponatremia are
Dilutional hyponatremia:
• If the urine specific gravity is <1.003, look for causes of excess free water administration by taking a careful history: inappropriate oral or intravenous (IV) hypotonic fluid administration in a patient with acute or chronic renal failure who cannot excrete free water maximally; low-solute
formulas or plain water in infants; excessive tap water use in infants with diarrhea or use as enemas; psychogenic polydipsia.
If none of these causes is present, use the urinary sodium concentration to help categorize the cause of hyponatremia:
Depletional hyponatremia with extrarenal losses:
• If the patient is HYPOVOLemic and the urinary sodium concentration is <20 mEq/L, the cause is likely to be gastrointestinal losses (vomiting, diarrhea, drainage tubes, fistulas, gastric drainage), skin losses (cystic fibrosis, heat stroke), or third spacing (burns, pancreatitis, muscle trauma,
effusions, ascites, peritonitis).
Depletional with renal losses:
• If the urine sodium concentration is >20 mEq/L, the cause is likely to be diuretics, osmotic diuresis, salt-losing nephritis, mineralocorticoid deficiency, congenital adrenal hypoplasia, pseudohypoaldosteronism.
• If the patient is EUVOLEMIC, and urine sodium concentration is >20 mEq/L, consider glucocorticoid or thyroid problems, syndrome of excessive antidiuretic hormone (SIADH), and the reset osmostat variant (a possible SIADH variant).
• If the patient is HYPERVOLemic and the urine sodium concentration is <20 mEq/L, consider edema-forming states: nephrotic syndrome, congestive heart failure, cirrhosis.
• If the urine sodium concentration is >20 mEq/L, consider acute or chronic renal failure.
Avner ED: Clinical disorders of water metabolism: hyponatremia and hypernatremia, Pediatr Ann 24:23–30, 1995.

KEY POINTS: DIFFERENTIAL DIAGNOSIS OF HYPONATREMIA
• Hyponatremia with an elevated serum creatinine suggests renal disease.
• Hyponatremia with high urine osmolality and high urine sodium suggests SIADH.
• Hyponatremia with hyperkalemia and metabolic acidosis suggests renal tubular hyperkalemia or a corticosteroid disorder.
• Hyponatremia with proteinuria and hypoalbuminuria occurs in nephrotic syndrome
SIADH, Syndrome of inappropriate antidiuretic hormone.

2. What is the emergency treatment of symptomatic hyponatremia?
Patients with central nervous system symptoms, particularly seizures, should receive an initial urgent intravenous infusion of hypertonic saline (3%) at a dose of 3 mL/kg. This should raise the serum sodium concentration by approximately 3 to 4 mEq/L. The dose can be repeated every 10 to

460

20 minutes. Increasing the serum sodium concentration by only 4 to 6 mEq/L is usually sufficient to stop hyponatremic seizures.

Brenkert TE, Estrada CM, McMorrow SP, et al: Intravenous hypertonic saline use in the pediatric emergency department,
Pediatr Emerg Care 29:71, 2013.

KEY POINTS: HYPONATREMIA
• Hyponatremia may reflect appropriate ADH stimulation as in cardiac decompensation and a non–steady state with arterial underfilling.
• Hyponatremia may reflect inappropriate ADH stimulation with euvolemia as in pneumonia, brain tumor, or severe pain.
• In children in intensive care and postoperatively, administration of hypotonic maintenance fluids increases the risk of hyponatremia when compared with administration of isotonic fluids.
• Isotonic fluids (normal saline) administered as maintenance should be avoided in children with ESRD, congestive heart failure, or hypertension.
ADH, Antidiuretic hormone.

3. How is the cause of hypernatremia established?
Hypernatremia is either due to excess salt administration or excess free water loss. A combination of history, clinical assessment of the patient’s volume status, and measurement of urine sodium concentration measurement is required to establish the diagnosis.
• If the patient is HYPOVOLemic and urine sodium concentration is <20 mEq/L, consider extrarenal water losses—diarrhea, excessive perspiration.
• If the urinary sodium concentration is >20 mEq/L, consider renal losses—renal dysplasia, obstructive uropathy, and osmotic diuresis.
• If the patient is EUVOLEMic and the urine sodium concentration is VARIABLE, consider extrarenal losses (insensible: dermal, respiratory) and renal losses (central diabetes insipidus, nephrogenic diabetes insipidus).
• If the patient is HYPERVOLEMic and the urine serum concentration is [usually] >20 mEq/L, consider— improperly mixed formula in tube feeding, excess sodium bicarbonate administration, excess salt administration, salt poisoning, and primary hyperaldosteronism (rare in children).

Avner ED: Clinical disorders of water metabolism: hyponatremia and hypernatremia, Pediatr Ann.24:23–30, 1995

4. Why can correcting hypernatremia too rapidly cause seizures?
Children with severe hyponatremia may seize before treatment is started, whereas those with hypernatremia may develop seizures in response to therapy. In patients with hypernatremic dehydration, increased extracellular osmolality draws fluid from the intracellular compartment, and cells, especially brain cells, shrink in size. However, the brain can generate idiogenic osmoles to minimize the loss of fluids. Idiogenic osmoles are principally amino acids and other organic solutes that allow brain cells to minimize cellular water loss. In fact, in chronic hypernatremia, brain size is almost normal. However, it takes about 24 hours to begin to generate or dissipate these idiogenic osmoles. Therefore, if correction of chronic hypernatremia of greater than 24 hours’ duration is too rapid, water moves from the extracellular compartment back into the cerebral intracellular compartment, thereby causing cerebral edema. This can lead to seizures, cerebral hemorrhage, and even death. To prevent this in patients with chronic hypernatremia, the serum sodium concentration should not be allowed to decrease
faster than 0.5 mEq/L per hour and ideally not more than 10 to 12 mEq/L/24 hr.

Schwaderer AL, Schwartz GJ: Treating hypernatremic dehydration, Pediatr REV 28:148–150, 2005.

5. What is the differential diagnosis of nephrogenic diabetes insipidus (NDI)?
• Inherited NDI may be due to arginine vasopressin (AVP) receptor (AVPR2 gene, X-linked) or aquaporin (AQP2 gene, recessive) abnormalities.
• Acquired NDI may be due to electrolyte causes (hypokalemia, hypocalcemia), medications (diuretics, lithium, cisplatin), chronic kidney diseases, tubulointerstitial disease.
• NDI can occur in renal Fanconi syndrome, renal tubular acidosis, and Bartter syndrome because of hypokalemia.

6. An infant boy presents with severe dehydration, polyuria, and hypernatremia. What is the first renal diagnosis that should come to mind?
Congenital NDI, which is caused by mutations in the aquaporin genes (AVPR2 or AQP2) is the first renal diagnosis that should come to mind. Aquaporins are membrane proteins involved in water transport. The genetic defect results in insensitivity in the distal nephron to AVP (ADH), so there is abnormal water reabsorption in the collecting ducts. This urine concentrating defect is present from birth, and symptoms of irritability, poor feeding, and poor weight gain begin in the first weeks
of life. High fevers, dehydration and seizures, mental retardation, and psychological problems can occur. Persistent polyuria can cause megacystis, trabeculated bladder, hydroureter, and hydronephrosis.

Wesche D, Deen PM, Knoers NV: Congenital nephrogenic diabetes insipidus: the current state of affairs, Pediatr Nephrol 27:2183–2104, 2012.

7. What are the clinical and physiologic consequences of progressive hypokalemia (low potassium)?
• Muscle weakness and paralysis, which can lead to hypoventilation and apnea
• Constipation and ileus
• Increased susceptibility for ventricular ectopic rhythms and fibrillation, especially in children receiving digitalis
• Interference with the ability of the kidney to concentrate urine, leading to polyuria

8. What are the causes of hypokalemia?
• Diuretics, occasionally laxatives
• Metabolic alkalosis, especially in patients with pyloric stenosis
• Severe diabetic ketoacidosis with dehydration
• Diarrhea
• Renal tubular acidosis, types I and II
• Renal Fanconi syndrome
• Bartter and Gitelman syndromes
• Hypermineralocorticoid states: Primary hyperaldosteronism, Cushing syndrome, adrenal tumors, rare forms of congenital adrenal hyperplasia, dexamethasone-suppressible hypertension
• Pituitary tumors producing adrenocorticotropic hormone
• Hyperreninemic states such as renal artery stenosis

9. Which foods are high in potassium?
Raisins, baked potatoes, cocoa, oranges, bananas, French fries (especially supersized!), and carrots are high in potassium.

10. List the causes of hyperkalemia in children.
• Increased potassium intake: Increased oral intake alone does not lead to hyperkalemia as long as the ability to excrete potassium is maintained. Increased intake is important when renal excretion is compromised in renal failure with oliguria-anuria or in patients taking angiotensin-converting enzyme (ACE) inhibitors. Rarely, an extremely high intake of potassium can lead to hyperkalemia (e.g., intravenous potassium, oral potassium penicillin, and blood transfusion using blood that has been stored a long time).
• Decreased renal excretion: Impaired renal function leads to reduced potassium excretion, usually in patients with oliguric/anuric acute renal failure. Initially, potassium balance is maintained by increased excretion through the functioning nephrons, until the glomerular filtration rate (GFR) decreases to <15 mL/min/1.73 m2.
• Redistribution of potassium from the intracellular to extracellular compartment. Metabolic
acidosis results in the movement of hydrogen ions into the intracellular space in order to buffer the intravascular pH. To maintain electroneutrality, potassium moves out of the cell resulting in hyperkalemia. Insulin promotes movement of potassium into cells. Therefore, insulin deficiency in diabetic ketoacidosis can lead to hyperkalemia. Intravascular hyperosmolality causes water movement out of cells, dragging potassium with it (solvent drag) followed by an increase in intracellular potassium concentration, creating a favorable gradient for potassium movement out of cells.

• Tissue breakdown can release potassium from the cells into the extracellular fluid. This occurs with trauma, rhabdomyolysis, chemotherapy (causing tumor lysis syndrome), massive hemolysis (e.g., transfusion reaction), strenuous exercise, and hyperkalemic periodic paralysis.
• Medication-induced hyperkalemia: β-blockers, potassium-sparing diuretics, ACE inhibitors, digoxin, succinylcholine, arginine, nonsteroidal anti-inflammatory drugs (NSAIDs), and calcineurin may induce hyperkalemia.
• Pseudohyperkalemia is defined as a rise in the serum potassium concentration with a concurrently normal plasma potassium concentration. The destruction of erythrocytes following venipuncture or capillary sampling is the most frequent reason for a raised serum potassium result in children. Pseudohyperkalemia is also seen with severe thrombocytosis due to release of potassium from platelet granules, severe polycythemia, or leukocytosis. In these instances, checking a plasma (as compared to a serum) potassium concentration will provide reassurance that the plasma concentration is normal.
• Aldosterone deficiency or resistance (pseudohypoaldosteronism) reduces potassium and hydrogen excretion and results in hyperkalemia and metabolic acidosis. Lack of aldosterone production occurs in primary adrenal insufficiency or with inborn errors of adrenal steroid metabolism (e.g., congenital adrenal hyperplasia, aldosterone synthase deficiency). Children with pseudohypoaldosteronism exhibit elevated aldosterone levels. Pseudohypoaldosteronism can occur with or without salt wasting.

Masilamani K, van der Voort J: The management of acute hyperkalemia in neonates and children, Arch Dis Child. 97:376–380, 2012.

11. When are calcium infusions indicated in a patient with elevated serum potassium?
If the serum potassium level is >8 mEq/L or there is a cardiac dysrhythmia, calcium infusions are indicated. A calcium infusion is the most rapid way to treat a dysrhythmia associated with hyperkalemia, but it does not reduce serum potassium concentrations. Hyperkalemia increases the cell’s membrane potential, thereby making cells more dysrhythmogenic. Hypercalcemia increases the cell’s threshold potential, restores the voltage difference between these two potentials, and decreases the likelihood of a dysrhythmia. The effect of a calcium infusion is transient.
12. What are key aspects in the treatment of hyperkalemia?
1. Stabilize membrane potentials: Ten percent calcium gluconate is used, most typically when immediate action is required to improve an abnormal electrocardiogram (ECG).
2. Induce potassium transport into cells: Therapies include glucose+ insulin and sodium bicarbonate (in the setting of acidosis). The use of beta-2 agonists (both intravenous and inhaled) are recommended by some experts as another possible therapy.
3. Enhanced excretion of potassium: This can be accomplished by a cation exchange resin (e.g., sodium polystyrene sulfonate); loop diuretic; and, as the ultimate therapy, dialysis.
13. What are the causes of periodic paralysis syndromes involving both high and low potassium levels?
Inherited channelopathies are disorders produced by abnormal ion channel function. In hypokalemic periodic paralysis, about 70% of patients have a mutation in a calcium channel gene. In hyperkalemic periodic paralysis, most cases are caused by mutations in the sodium channel SCN4A. Both diseases are characterized by intermittent weakness, usually in the morning. Of note, hyperkalemic period paralysis is a well-described and common malady in quarter horses, where it is called “Impressive syndrome” after the mutational source was found to have originated in a stallion named Impressive.

Saperstein DS: Muscle channelopathies, Semin Neurol 28:260–269, 2008.

14. What is the undetermined serum anion gap?
The undetermined serum anion gap is the difference between the serum sodium concentration and the sum of chloride plus bicarbonate. The anion gap represents anions that are not normally measured such as sulfate, organic anions, and charged albumin. The normal value is
<15 mEq/L.

15. How is the serum anion gap helpful in the evaluation of a metabolic acidosis? In the presence of metabolic acidosis, the calculation of the anion gap determines which of two diagnostic pathways is more likely. If the anion gap is increased, consider a cause listed under MUDPILES (see question 16). If the anion gap is normal, consider diarrhea or renal tubular acidosis. Of note, always suspect an undetermined anion gap acidosis if the serum chloride and bicarbonate are both low.

16. What are the causes of an elevated serum anion gap acidosis?
An increased anion gap reflects the addition of an acid with its anion that is not normally measured, such as salicylate but not hydrochloric acid. The mnemonic MUDPILES helps with remembering the causes of an elevated anion gap.
• Methanol (formic acid and formate)
• Uremia (guanidinosuccinic acid, phosphates, sulfates, and other acids)
• Diabetic ketoacidosis (lactic acid, β-hydroxybutyrate, and acetoacetate)
• Paraldehyde, Phenformin
• Iron, Isoniazid, Inborn errors of metabolism
• Lactic acidosis secondary to hypoxia, severe cardiorespiratory depression, shock, prolonged seizures or mitochondrial diseases
• Ethanol, Ethylene glycol
• Salicylate

17. How limited is the respiratory response to metabolic alkalosis? Metabolic alkalosis occurs when a net gain of alkali or loss of acid leads to a rise in the serum bicarbonate concentration and pH. In metabolic alkalosis (as in metabolic acidosis), there is a measure of respiratory compensation in response to the change in pH. This response, which is accomplished by alveolar hypoventilation, is limited by the overriding need to maintain an adequate blood oxygen concentration.
Usually the PCO2 will not increase >50 to 55 mm Hg despite severe alkalosis.
18. What is the differential diagnosis in a child presenting with symptoms of primary metabolic alkalosis?
Metabolic alkalosis can be divided into two major categories on the basis of the urinary chloride concentration and the response to VOLUME expansion with a saline infusion.

Saline-responsive metabolic alkalosis: The urine chloride concentration is <10 mEq/L and there is significant volume depletion. Intravenously administered normal saline usually corrects the metabolic alkalosis; the classic example is pyloric stenosis.
Causes include pyloric stenosis, profuse vomiting, excessive upper gastrointestinal suctioning, congenital chloride diarrhea, laxative abuse, diuretic use or abuse, cystic fibrosis, chloride-deficient infant formulas, post-hypercapnia syndrome, and administration of poorly reabsorbable anion.
This can also occur after treatment of organic acidemias. Treatment of diabetic ketoacidosis with insulin leads to metabolism of acetoacetate, which results in the generation of bicarbonate.
Saline-resistant alkalosis: The urine chloride is high and the patient is hypertensive. Administration of normal saline aggravates the metabolic alkalosis. In most of these cases, mineralocorticoid excess plays the central role in the generation of the alkalosis.
Causes include hyperreninemic hypertension (renal artery stenosis, renin-secreting tumor), corticosteroid treatment, severe potassium deficiency, genetic block in steroid hormone synthesis (17α-OH deficiency, 11β-OH deficiency), Liddle syndrome, Bartter syndrome, Gitelman syndrome, primary hyperaldosteronism (extremely rare in children), and licorice-containing glycyrrhizic acid.

19. Why is the urine pH often acidic (pH 5.0 to 5.5) in a child with metabolic alkalosis from severe vomiting?
Prolonged vomiting, such as is seen in pyloric stenosis, results in metabolic alkalosis because of loss of hydrogen ions and volume depletion (dehydration). There is also significant sodium, potassium, and chloride loss with a resultant hypokalemic, hypochloremic metabolic alkalosis. The volume depletion activates the renin-angiotensin-aldosterone response which results in increased distal reabsorption of sodium and water. To retain sodium, the kidney must release other cations (hydrogen in particular) into the urine. The hydrogen ions lower the urine pH. When volume is repleted, there will be suppression of aldosterone, the urine pH will become alkaline (pH 6.5 or more), and the metabolic alkalosis will lessen. The change of the urine from acidotic to alkalotic is one sign of adequate volume replenishment.

This scenario is sometimes referred to as the “paradoxical aciduria of metabolic alkalosis,” and your attending may use the term as well. If you are feeling courageous, you could respond that it is not paradoxical at all, once you understand the pathophysiology.

ACUTE KIDNEY INJURY
20. Why has the term acute kidney injury replaced acute renal failure? Acute kidney injury (AKI) reflects more appropriately the concept that smaller reductions in kidney function (short of complete organ failure) have significant clinical repercussions in terms of morbidity and mortality.
21. What clinical and laboratory observations are useful for distinguishing prerenal oliguria (decreased effective circulating volume) from the oliguria of intrinsic AKI? Clinical assessment of hydration, volume, and perfusion status is critical because these are more likely to be impaired in a prerenal state. In patients with intrinsic AKI there is more likely to be normal or excess volume; there may be evidence of edema or vascular congestion. If the assessment of the volume status suggests a volume deficit, a fluid bolus with normal saline can be both diagnostic
and therapeutic. Laboratory studies that assist are summarized in Table 12-1.

Table 12-1. Laboratory Studies That May Distinguish Prerenal Oliguria From Acute Tubular Necrosis
PARAMETER PRERENAL OLIGURIA RENAL OLIGURIA
Random UNa (mEq/L) <20 >40
FENa* <1% >1%
Urine osmolality (m0sm/L) >500 <300
FENa ¼([UNa × PCreat]/[PNa × UCreat])× 100% (on a randomly collected, spot urine).

22. What is the most common cause of AKI in young children in the United States? This used to be the hemolytic uremic syndrome (HUS), which in most cases is caused by gastrointestinal infection with Shiga toxin–producing E. coli, especially the O157:H7 serotype. However, cases of acute tubular necrosis in infancy and childhood from hypoxic, hypotensive, and/or hypovolemic insults, or drug-induced injury now represent the largest group of causes. New observations indicate that the presence of proteinuria predicts the development of AKI independently of eGFR (estimated glomerular filtration rate based on creatinine level). The severity of AKI and duration are important predictors of chronic kidney disease and long-term mortality.
The causes of AKI are listed in Table 12-2.

Siew ED, Furth SL: Acute kidney injury: a not-so-silent disease, Kidney Int 85:494–495, 2014.
Siew ED, Deger SM: Recent advances in acute kidney injury epidemiology, Curr Opin Nephrol Hypertens 21:309– 317, 2012.

Table 12-2. Causes of Acute Kidney Injury (AKI)
Prerenal Causes of AKI
Volume depletion Severe diarrhea
Protracted vomiting
Osmotic diuresis
Diuretics
Extensive burns
Hemorrhage
Decreased effective blood volume Septic shock Anaphylaxis Nephrotic syndrome
Continued on following page

Table 12-2. Causes of Acute Kidney Injury (AKI) (Continued )
Cardiac failure Anatomical malformation
Arrhythmias
Cardiomyopathy
Tamponade
Post-cardiac surgery
Intrinsic Causes of AKI
Postinfectious glomerulonephritis
Lupus nephritis
Henoch-Sch€onlein purpura nephritis
IgA nephropathy
Crescentic glomerulonephritis
Vascular Renal venous thrombosis Vasculitis
Nonsteroidal anti-inflammatory agents ACE inhibitors
Hemolytic uremic syndrome
Tubular (ATN) Severe prerenal failure
Asphyxia/hypoxemia
Crystal obstruction
Medications
Toxins
Tumor-lysis syndrome
Interstitial nephritis Allergic interstitial nephritis
TINU syndrome
Malignancy infiltrate
Pyelonephritis
Sarcoidosis
Postrenal Causes of AKI
Bilateral nephrolithiasis Neoplasm
ACE ¼ angiotensin-converting enzyme; ATN ¼ acute tubular necrosis; IgA ¼ immunoglobulin A; TINU ¼ tubulointerstitial nephritis and uveitis.

23. What is the triad of clinical findings of HUS?
• Acute renal failure with oliguria, anuria, and rarely polyuria
• Acute hemolytic anemia: microangiopathic with fragmented red blood cells (RBCs) or schistocytes, nonimmune, Coombs negative
• Thrombocytopenia

Kaplan BS, Drummond KN: The hemolytic-uremic syndrome is a syndrome, N Engl J Med 298:964–966, 1978.

24. What are the causes of HUS?
Shiga toxin–producing E. coli OH157:H7 is the most frequent cause of HUS in the United States. Shiga toxin–producing E. coli O104:H4 infection caused a severe epidemic of HUS in Europe in 2011. Endothelial cell injury with secondary glomerular capillary microthrombi is central to the pathogenesis of HUS caused by Shiga toxin–producing E. coli. This is the most frequent cause of HUS, but there are other known causes of which atypical hemolytic uremic syndrome (aHUS) comprises about 10% of all causes. Genetic mutations increase the risk of aHUS and may lead to uncontrolled activation of the complement system when it is triggered.

25. Does the use of antibiotic therapy in children with diarrhea caused by E. coli OH157:H7 prevent HUS?
This is controversial. A study showed that children who received antibiotics (usually sulfa-containing or β-lactam antibiotics) during outbreaks had a higher rate (50% versus 7%) of HUS. A subsequent meta-analysis showed no protection and no association. Most specialists opt not to treat patients with E. coli OH157:H7 gastroenteritis with antibiotics because no benefit has been proven.

Safdar N, Said A, Gangnon RE, et al: Risk of hemolytic-uremic syndrome after antibiotic treatment of Escherichia coli
O157:H7 enteritis: a meta-analysis, JAMA 288:996–1001, 2002.

26. Can anything be done to lessen the severity of the renal disease in children with HUS caused by E. coli OH157:H7?
About 15% of patients with E. coli 0157:H7 gastroenteritis develop HUS within 2 to 14 days of onset of diarrhea. Intravenous volume expansion during this period, if indicated, may decrease the frequency of oligoanuric renal failure in patients with E. coli OH157:H7 at risk for HUS.
Additional therapies, such as plasma exchange, immunoadsorption, Shiga toxin–binding agents and complement inhibitors (e.g., eculizumab) are currently under study.

Keir LS, Marks SD, Kim JJ: Shigatoxin-associated hemolytic-uremic syndrome: current molecular mechanisms and future therapies, Drug Des DEVEL Ther 6:195–208, 2012.
Hickey CA, Beattie TJ, Cowieson J, et al: Early volume expansion during diarrhea and relative nephroprotection during subsequent hemolytic uremic syndrome, Arch Pediatr Adolesc Med 165:884–889,2011.

27. Why is Shiga toxin–associated HUS so terrifying for patients, family, and physicians?
Shiga toxin–associated HUS is terrifying because patients can die, intensive care is required for about 50% of cases, serious extrarenal complications can develop, and patients who recover may have chronic sequelae. Fortunately, about 70% of patients recover completely from the acute episode. The acute death rate is now <4%, and serious long-term complications are <15%. The kidneys bear the brunt of the long-term damage, which includes proteinuria (15% to 30% of cases),
hypertension (5% to 15%), chronic kidney disease (CKD, 9% to 18%), and end-stage renal disease (ESRD) (3%). A smaller number have extrarenal sequelae, including colonic strictures, cholelithiasis, diabetes mellitus, or brain injury. Most of the patients who progress to ESRD do not recover normal renal function after the acute episode. The most important risk factors for both poor acute and long-term renal outcome are anuria for >10 days and prolonged need for dialysis >3 weeks. After the acute
episode, all patients must be followed for at least 5 years and patients should be followed
indefinitely if there is proteinuria, hypertension, or a reduced eGFR.

Spinale JM1, Ruebner RL, Copelovitch L, Kaplan BS: Long-term outcomes of Shiga toxin hemolytic uremic syndrome,
Pediatr Nephrol 28:2097–2105, 2013.

28. What is meant by atypical hemolytic uremic syndrome (aHUS)? This term describes a group of patients of all ages who present with the classic features of HUS but who do not HAVE Shigalike–toxin producing E. coli (STEC) as the cause. Atypical HUS accounts for about 5% to 10% of HUS cases. Several genetic mutations have been identified that appear to cause excessive activation of the complement system. The classical distinctions between so-called typical and atypical HUS are starting to blur with new discoveries in abnormalities in genes that regulate the alternate pathway of complement. Many cases of aHUS may even have antecedent diarrhea. Compared with classic HUS, the prognosis for aHUS is worse. Fifty percent may progress to ESRD compared with 85% of cases of typical HUS who recover renal function. The use of eculizumab, a specific anti- C5 monoclonal antibody that blocks alternative complement pathway activation, has dramatically improved the outcome of aHUS patients with known or suspected mutations in complement regulatory genes.

Legendre CM, Licht C, Muus P, et al: Terminal complement inhibitor eculizumab in atypical hemolytic-uremic syndrome,
N Engl J Med 368:2169–2181, 2013.
Kaplan BS, Ruebner RL, Copelovitch L: An evaluation of the results of eculizumab treatment of atypical hemolytic uremic syndrome, Expert Opin Orphan Drugs 2:167–176, 2013.

29. What are indications for dialysis in AKI?
A helpful pneumonic is AEIOU: Acidemia, Electrolyte abnormalities, Increased blood pressure,
Overload (volume), and Uremia
• Severe metabolic acidemia that cannot be controlled with sodium bicarbonate
• Elevated blood urea nitrogen (BUN) and creatinine concentrations in the context of anuria or uncontrolled metabolic abnormalities. There are no established critical levels of BUN or
creatinine above which dialysis needs to be instituted. However, when the creatinine reaches 10 mg/dL or the BUN is 100 mg/dL, the GFR is usually markedly reduced, and this results in one or more of the following abnormalities:
• Hyperkalemia that is either rapidly rising or stable at a dangerously high level, especially with ECG changes, that is not controlled by infusions of insulin, bicarbonate, calcium,
or Kayexalate-binding resin; other severe electrolyte disturbances, including symptomatic hyponatremia, hypocalcemia, hyperphosphatemia, and hyperuricemia
• Volume-dependent hypertension or signs of CHF not responsive to diuretic treatment
• Urgent need for a blood transfusion in the presence of fluid overload and/or hypertension
• Acute signs or symptoms of encephalopathy

CHRONIC KIDNEY DISEASE
30. Based on GFR estimations and serum creatinine levels, how are AKI and chronic kidney disease (CKD) defined?
Criteria for AKI: GFR <60 mL/min per 1.73 m2 for <3 months, OR decrease in GFR by >35% or increase in serum creatinine (SCr) by >50% for <3 months
Criteria for CKD: Presence of AKI criteria for >3 months
Kidney Disease: Improving Global Outcomes (KDIGO) CKD Work Group: KDIGO 2012 clinical practice guideline for the evaluation and management of chronic kidney disease, Kidney Inter 3S:1S–150S; 2013.

31. What are the stages of CKD?
There are 5 stages of CKD (Table 12-3). The stages are based on GFR.

Table 12-3. Stages of Chronic Kidney Disease
STAGE GFR*
DESCRIPTION TREATMENT
1 90 + Normal kidney function but urine or imaging abnormalities Observation, control blood pressure
2 60-89 Mildly reduced function Observation, control blood pressure control
3A
3B 45-59
30-44 Moderately reduced function Observation, control blood pressure and risk factors
4 15-29 Severely reduced function Plan for end stage renal failure
5 < 15 or on dialysis End-stage kidney disease Discuss treatment choices
*GFR= Glomerular filtration rate (min/1.73 m2).
Kidney Disease: Improving Global Outcomes (KDIGO) CKD Work Group: KDIGO 2012 Clinical Practice Guideline for the Evaluation and Management of Chronic Kidney Disease, Kidney Int 3S:1–150; 2013.

32. What are the main causes of CKD in children that result in renal transplantation?
• Obstructive uropathy
• Aplastic, hypoplastic, and dysplastic kidneys
• Focal segmental glomerulosclerosis

Whyte DA, Fine RN: Chronic kidney disease in children, Pediatr REV 29:335–340, 2008.

33. Your nephrology attending likes to use the Socratic method of teaching. “What are four major endocrine and cardiac medications that have dramatically improved the lives and outcomes of children with CKD?” is a favorite question.
• Use of erythropoietin, a hormone normally produced in the kidney, has largely eliminated the need for blood transfusions in children with CKD.
• Use of 1,25-dihydroxycholecalciferol, an active form of vitamin D normally produced in the kidney, has dramatically prevented or treated osteodystrophy.
• Growth hormone (GH) is not produced by the kidney. However, recombinant GH administration results in growth acceleration in growth-retarded children with CKD.
• Inhibition of the renin-angiotensin-aldosterone system with ACE inhibitors or angiotensin receptor blockers (ARBs) has helped control hypertension and also prevented progression of glomerular fibrosis.

34. What is the new term for renal osteodystrophy?
Renal osteodystrophy has traditionally been the term used to describe the bone and mineral pathology caused by the endocrine and electrolyte derangements in CKD. The moniker “chronic kidney disease–mineral and bone disorder (CKD–MBD)” was recommended by an international consensus committee in 2009 to better describe the systemic changes that take place in CKD.

35. What hormonal elevation is key in the pathogenesis of CKD–MBD? Hyperparathyroidism is key. Decreased GFR results in retention of phosphate and hyperphosphatemia. Additionally, there is decreased renal 1,25-dihydroxyvitamin D production. These two factors lead to decreased absorption of calcium from the gastrointestinal tract and decreased responsiveness of bone to parathyroid hormone (PTH), which results in hypocalcemia. The hypocalcemia leads to an increased release of PTH, which then increases bone resorption. A long-term sequela of this secondary hyperparathyroidism from CKD can be bone marrow fibrosis.

36. Why is it important to recognize CKD–MBD at an early stage?
Recognition of osteodystrophy, (which begins when the GFR is half of the normal rate) is important because early intervention with calcitriol, vitamin D, and phosphate binders can prevent and/or heal the bone disease (but not necessarily enhance growth). Furthermore, in states of chronic acidosis, the skeleton acts as a buffer for the net acid retained. This results in the release of calcium, which contributes to further osteopenia and bone disease.

37. Why is FGF23 an important evolving concept in CKD–MBD?
Changes in calcium/phosphate metabolism in CKD are characterized by hyperphosphatemia, hypocalcemia, calcitriol deficiency, and hyperparathyroidism. A key regulator of this complex system is FGF23, a circulating peptide produced in the osteocyte that regulates renal phosphate excretion. PTH is still the most important biomarker, but novel biomarkers, such as FGF23, and noninvasive imaging techniques are emerging that can allow for individual classification and monitoring in progression of CKD-MBD.

Kemper MJ, van Husen M: Renal osteodystrophy in children: pathogenesis, diagnosis and treatment, Curr Opin Pediatr 26:180–186, 2014.

38. What are the ciliopathies?
This is an important question because the answer opens up a whole new book in the understanding of many inherited renal conditions (Table 12-4). Diverse developmental and degenerative single-gene disorders such as polycystic kidney disease, nephronophthisis, retinitis pigmentosa, Bardet–Biedl syndrome, Joubert syndrome, and Meckel syndrome are categorized as ciliopathies, a recent concept that describes diseases characterized by dysfunction of a hairlike cellular organelle called the cilium. Most of the proteins that are altered in these single-gene disorders function at the level of the cilium–centrosome complex.

Hildebrandt F, Benzing T, Katsanis N: Ciliopathies, N Engl J Med 364:1533–1543; 2011.

Table 12-4. Prominent Single-Gene Ciliopathies
Dominant Disorders Autosomal Dominant Polycystic Kidney Disease Von Hippel–Lindau Disease
Recessive Disorders Autosomal Recessive Polycystic Kidney Disease Nephronophthisis
Bardet–Biedl syndrome
Retinal–Renal Syndromes
Senior–Løken syndrome).
Joubert Syndrome
Meckel Syndrome

39. Nephronophthisis is difficult to pronounce and spell, but what is it?
Nephronophthisis (one of the ciliopathies) is the most frequent genetic cause of ESRD in the first 3 decades of life (median age: 13 years) and is characterized by cysts restricted to the corticomedullary junction. Kidney size is normal or small. Mutations in more than 11 recessive genes (NPHP1 to NPHP11) have been identified as causes of nephronophthisis. Mutations in NPHP1 cause juvenile nephronophthisis type 1. Clinical features of juvenile nephronophthisis include anemia, polyuria, polydipsia, isosthenuria, growth failure, and progression to end stage kidney disease.
40. Which is more common in children: autosomal dominant polycystic kidney disease (ADPKD) or autosomal recessive polycystic kidney disease (ARPKD)? ADPKD is the more common, and it also is the most prevalent monogenic disorder in humans. Cysts are typically diagnosed incidentally (e.g., affected parent, imaging study for another reason). ADPKD may present with pain, hematuria, UTI, hypertension, and calculi and can even be diagnosed in utero. There is a large interfamilial and intrafamilial variation with genetic heterogeneity and modifier genes. Two polycystic kidney (PKD) genes have been identified: PKD1 (85% of cases) and PKD2 (15% of cases).
41. What anatomic features characterize ADPKD?
Bilateral renal cysts are the predominant feature, but involvement of other organs can include cysts (liver, seminal vesicles, pancreas, arachnoid); intracranial aneurysms; mitral valve prolapse; diverticulosis; and rarely, aortic root dilatation and aortic aneurysms.
42. How is ADPKD diagnosed and managed?
A renal ultrasound is usually adequate. Deoxyribonucleic acid (DNA) studies are rarely indicated. Presymptomatic diagnosis in children outweighs any benefits until effective treatments are available. It is important to counsel presymptomatic individuals before testing.
Monitor blood pressure and urine in individuals with a family history of ADPKD. Avoid contact sports. Use of ACE inhibitors to control blood pressure has improved outcomes. Encourage water intake to suppress antidiuretic hormone (ADH) and retard cyst growth. The prognosis is excellent in children.

Torres VE, Harris PC, Pirson Y: Autosomal dominant polycystic kidney disease, Lancet 369:1287–1301, 2007. Polycystic Kidney Foundation: www.PKDcure.org. Accessed on Mar. 20, 2015.
PKD Alliance: www.arpkdchf.org. Accessed on Mar. 20, 2015.

43. Why was the term infantile polycystic kidney disease replaced with ARPKD?
This is because some patients have been diagnosed in adulthood with moderate renal insufficiency and ESRD. The characteristic dilatation of the renal collecting ducts begins during development and can present at any stage from infancy to adulthood. Renal insufficiency often occurs in utero and may lead to early abortion or oligohydramnios and lung hypoplasia. However, there are affected
neonates who have no evidence of renal dysfunction. Up to 30% of patients die in the perinatal period, and those surviving the neonatal period can reach ESRD in infancy, early childhood, or adolescence. The clinical spectrum of ARPKD includes bilateral renal enlargement with microcysts, arterial hypertension, and intrahepatic biliary dysgenesis. Affected infants develop congenital hepatic

fibrosis and some have nonobstructive dilation of the intrahepatic bile ducts (Caroli disease). Cholangitis, variceal bleeding, and hypersplenism are serious complications. ARPKD is caused by mutations in the PKDHD1 gene on chromosome 6.

B€uscher R, B€uscher AK, Weber S, et al: Clinical manifestations of autosomal recessive polycystic kidney disease (ARPKD): kidney-related and non-kidney-related phenotypes, Pediatr Nephrol 29:1915–1925, 2014.

ENURESIS/DYSFUNCTIONAL VOIDING
44. How common is nocturnal enuresis in older children?
At the age of 5 years, about 20% of children (boys more than girls) wet the bed at least once monthly. Nightly wetting is not as common (<5%). By the age of 7 years, the overall rate is down to 10%, and by the age of 10 years, it is down to 5%. As a general rule, after age 7 years, nocturnal enuresis resolves at a rate of 15% per year so that by age 15 years, about 1% to 2% of teenagers still have nocturnal enuresis.
45. Why does nighttime bed-wetting persist in some children?
Ninety-seven percent or more of the causes are nonpathologic, and a number of explanations have been theorized: maturational delay of neurodevelopmental processes, small bladder capacity, genetic influences, difficulties with waking, and decreased nighttime secretion of ADH. No data support the belief that wetting occurs during “deep sleep.” Genetic influences are quite strong. If both parents were enuretic, a child’s likelihood is about 75%; if one parent was involved, the likelihood is about 50%. Psychological problems are unlikely to cause nocturnal enuresis, but they are more common if daytime symptoms are present.

Graham KM, Levy JB: Enuresis, Pediatr REV 30:165–172, 2009.

46. What treatments are available for nocturnal enuresis?
The therapeutic approach depends in large part on the age of the patient, the effect of the problem on the patient, and the parents’ attitude. It is important to realize that 15% of patients per year will spontaneously improve.

Dry bed training: Self-waking routines, cleanliness training, bladder training, and rewards for dry nights; generally not effective as a sole intervention
Enuresis alarms: Portable alarms (auditory and/or vibratory) worn by the child at night and designed to awaken the child to the sensation of a full bladder; success rates as high as 70%; safe, but requires parental and child motivation
Desmopressin: Synthetic analog of vasopressin that, at the renal level, increases distal tubular reabsorption of water, thus diminishing nighttime bladder volume; available in oral and nasal forms; up to 70% effective; high relapse rate after discontinuation (similar to placebo); possible adverse effects, including nasal irritation and hyponatremia; expensive
Imipramine: Bladder effects include increasing capacity and decreasing detrusor excitability; high relapse rate; important central nervous system side effects in 10% (e.g., drowsiness, agitation, sleep disturbances)
47. A 7-year-old presents with problems of intermittent daytime urinary incontinence with a normal urinalysis and a negative urine culture. What evaluation is needed? The differential diagnosis is broad with considerable clinical overlap, including problems with bladder storage (overactive bladder or urge syndrome) and dysfunctional voiding, in which the child habitually contracts the external urinary sphincter during micturition. Keys to diagnosis are a good history of the pattern and circumstances of the incontinence, urinalysis/urine culture, a bladder diary, uroflowmetry (and assessment of postvoid residual), and baseline renal ultrasonography with efforts to exclude any neurogenic, infectious, or anatomic abnormalities. The prevalence in school-age children of daytime incontinence is remarkably high, almost 1 in 5, in some studies. The problem can have profound psychosocial effects, so attempts at a diagnosis are key. Urology referral is often required.

Deshpande AV, Craig JC, Smith GHH, et al: Management of daytime urinary incontinence and lower urinary tract symptoms in children, J Paed Child Health 48:e44–e52, 2012.

48. What is the term for extraordinary daytime urinary frequency? Pollakiuria is characterized by a very high daytime frequency of micturition (as high as 50 times per day). Symptoms are limited to the daytime. It is seen around 4 to 6 years in either gender and is associated with a history of recent death or life-threatening event in the family. It usually runs a benign, self-limiting course over 6 months. No specific treatment, apart from reassurance, is necessary. Children presenting with frequency, however, merit clinical investigation to exclude other pathologic causes.
49. Why is giggle incontinence not a laughing matter?
This uncommon form of daytime incontinence usually occurs in school-age girls. There is moderate to large amounts of urinary leakage triggered by laughing. The accepted theory is that of a central inactivation (cataplexy) in association with laughter resulting in incontinence. It is a diagnosis of exclusion and is usually established on history and is supplemented by the absence of other voiding symptoms and normal investigations. Giggle incontinence has a significant adverse effect on social life, and this is often why medical assistance is sought.
50. What is the normal bladder capacity in children? Bladder capacity is a reflection of voided volumes and is an important factor in the evaluation of children with voiding dysfunction. It is estimated (in mL) by the formula: [30 +(age in years× 30)]. The formula is useful up to age 12 years, after which age the estimated bladder capacity is 390 mL (an approximate adult value).

Nevéus T, von Gontard A, Hoebeke P, et al: The standardization of terminology of lower urinary tract function in children and adolescents: report from the Standardisation Committee of the International Children’s Continence Society, J Urol 176: 314–324, 2006.

GLOMERULAR DISEASES
51. What constellation of findings defines nephrotic syndrome?
The nephrotic syndrome is defined by proteinuria, hypoalbuminemia, edema, and hypercholesterolemia. There are, however, patients with nephrotic-range proteinuria and mild-to- moderate hypoalbuminemia or even normal albumin levels in whom there is no hyperlipidemia or peripheral edema. A proportion of such patients have focal segmental glomerulosclerosis (FSGS).
52. What differentiates nephrotic syndrome from nephritis?
The suffix “-itis” implies evidence of glomerular inflammation. On biopsy, this is an increased number of the cells within the glomerulus and/or the presence of leukocytes. Glomerular inflammation disrupts glomerular basement membrane structure and function and leads to hematuria and proteinuria. The proteinuria may be minimal to massive, depending on the type and severity of the nephritis. The finding of RBC casts (Fig.12-1) in the urine is, with rare exceptions, diagnostic of glomerulonephritis.

Figure 12-1. Red blood cell cast from a patient with streptococcal glomerulonephritis. These casts are almost always associated with glomerulonephritis or vasculitis and virtually exclude extrarenal disease. (From Zitelli BJ, DAVIS HW: Atlas of Pediatric Physical Diagnosis, ed 4. St. Louis, 2002, Mosby, p 458.)

Nephrosis is another term for the nephrotic syndrome. There are many causes of the nephrotic syndrome, and confusingly several different histopathological changes even with the same cause. Lupus nephritis is a classic example. Not only are there several causes of lupus (e.g., genetic, drugs such as hydralazine) but also at least five classes of renal pathologic changes. Most glomerulopathies present with a nephritic or a nephrotic syndrome. Many patients present with a mixed picture of nephritic/nephrotic syndrome.
53. A 12-year-old girl complains of a sore throat and passes painless coke-colored urine for 2 days. When you see her a few days later she has amber colored urine, microscopic hematuria, 2 + proteinuria, RBC casts, and no other complaints or clinical findings. What is the most likely cause of this presentation?
IgA nephropathy (IgAN) is by far the most likely cause. The simultaneous occurrence of upper respiratory symptoms and gross hematuria make poststreptococcal glomerulonephritis less likely. IgAN has a more benign clinical course in children than adults; pediatric patients are more likely to have minimal histologic lesions and less likely to have advanced chronic lesions. IgAN is the most common type of primary glomerular disease worldwide. The clinical and histological features of IgAN are variable and include microscopic hematuria, synpharyngitic hematuria (i.e., hematuria after upper respiratory infection [URI]), recurrent hematuria, proteinuria, nephrotic syndrome, nephritic syndrome, and acute renal failure. Glomerular deposits of IgA characterize IgAN (Fig. 12-2). There is no proven therapy, but ACE inhibitors may retard or prevent sclerosis.

Figure 12-2. Diffuse mesangial IgA nephropathy is seen on indirect immunofluorescence with fluorescein isothiocyanate—anti-IgA. (From Johnson RJ, Feehally J, Floege J: Comprehensive Clinical Nephrology, ed 5. Philadelphia, 2015, Saunders, pp 266–277.)

54. What is the long-term prognosis for patients with IgAN?
IgAN is the most common form of glomerulonephritis that results in ESRD. Twenty percent to 25% of patients will progress to ESRD over 25 years. Risk factors for developing ESRD include elevated serum creatinine, proteinuria of 1 g/day, hypertension and the severity of interstitial fibrosis, tubular atrophy, and the extent of glomerular sclerosis.

Haas M: IgA nephropathy in children and adults: comparison of histologic features and clinical outcomes, Nephrol Dial Transplant 23:2537–2545, 2008

55. During the evaluation of a patient with hematuria, what features suggest acute glomerulonephritis, chronic glomerulonephritis, or nephrotic syndrome?
The three major presentations of glomerular involvement are
• Acute glomerulonephritis: Edema, proteinuria of 1 + or greater, hypertension, oliguria, dysmorphic RBCs, or RBC casts on urinalysis
• Chronic glomerulonephritis: Minimal acute symptoms; may have chronic fatigue, failure to thrive, normochromic normocytic anemia, hypertension, abnormal urinalysis, high BUN and creatinine concentrations (azotemia), metabolic acidosis, hypocalcemia, and hyperphosphatemia
• Nephrotic syndrome: Proteinuria of > 40 mg/m2 per hour, edema, hypoalbuminemia, and hypercholesterolemia

56. If glomerulonephritis is suspected, what laboratory tests should be considered?
First-line tests: dipstick urinalysis; urine microscopy; serum electrolytes; BUN and serum creatinine; serum C3 and C4; streptococcal serology (anti-streptolysin O titer or Streptozyme),throat culture; skin culture if impetigo is present; serum albumin
Second-line tests: antinuclear antibody (ANA), anti-DNA antibodies (if lupus nephritis is suspected); hepatitis B and C serology (for patients in endemic areas, those previously transfused or individuals who engage in high-risk behavior); antineutrophil cytoplasmic antibody (ANCA)
(if rapidly progressive glomerulonephritis or vasculitis is suspected)
57. Which glomerulonephritides have a genetic basis?
See Table 12-5.

Table 12-5. Glomerular Diseases With Genetic Causes
MUTATION INHERITANCE
Congenital nephrotic syndrome NPHS1 Recessive
Diffuse mesangial sclerosis WT1 Recessive
Denys-Drash syndrome WT1 Recessive
Frazier syndrome KTS Recessive
FSGS NPHS2, TRPC6, ACTN4, INF2
and PLCE1 Recessive or dominant
Alport syndrome COL4A5 X-linked Recessive,
dominant
Nail-patella syndrome LMX1B Dominant
Steroid resistant NS with sensorineural deafness COQ6 Dominant
C3 glomerulopathy factor H, CFHR Recessive
Hildebrandt F: Genetic kidney diseases, Lancet 375;1287-1295, 2010.
Carney EF: Glomerular disease: Frequency of podocyte-related gene mutations in FSGS, Nat REV Nephrol 10:184, 2014.

58. Which types of glomerulonephritis are associated with hypocomplementemia?
• Postinfectious glomerulonephritis, including poststreptococcal (ASPGN); staphylococcus in subacute bacterial endocarditis
• Lupus nephritis
• Membranoproliferative glomerulonephritis
• C3 glomerulopathy
• aHUS
59. Does the treatment of streptococcal skin or pharyngeal infections prevent ASPGN?
No. Treatment of impetigo or pharyngitis does not prevent glomerulonephritis in the index case. However, treatment lessens the likelihood of contagious spread to children who may be susceptible.
60. What is the usual time course for ASPGN?
Symptoms and signs begin about 7 to 14 days after pharyngitis and as long as 6 weeks after a pyoderma with Lancefield group A β-hemolytic streptococci. Children typically have tea-colored urine and edema. The acute phase (hypertension, gross hematuria, oliguria) can last as long as
3 weeks. Serum C3 levels may remain depressed for up to 8 weeks, but persistence beyond this point suggests another diagnosis. Chronic microscopic hematuria can persist for up to 2 years. In pediatric patients, full recovery is expected, and progression to chronic renal insufficiency is rare.

61. What percentage of children with ASPGN have elevated levels of serum anti-streptolysin O titers?
About 80% to 85% of children with documented pharyngeal streptococcal infections develop elevated antistreptolysin (ASO) titers. Streptolysin O is bound to lipids in the skin, so that the percentage of individuals with streptococcal impetigo who develop positive ASO titers is much lower. For this reason, a normal ASO titer does not rule out recent streptococcal infection. Screening for other streptococcus-associated antigens, antihyaluronidase, and anti-DNAase B titers or the use of the Streptozyme test, which measures four of the streptococcal antigens, will be positive in more than 95% of children with documented streptococcal infection.

62. If pharyngitis and the brown urine occur on the same day or within 1 or 2 days, does this make ASPGN less likely?
Yes. The occurrence of upper respiratory symptoms and gross hematuria at the same time (synpharyngitic) is more characteristic of IgA nephropathy (IgAN). Serum C3 is normal in IgAN. These children may have recurrent episodes of synpharyngitic (i.e., at the time of or shortly after a URI) gross hematuria.

63. A 7-year-old girl has a typical presentation of poststreptococcal glomerulonephritis with positive ASO titers and low serum C3, but over the next 3 months she has recurrent episodes of gross hematuria and her C3 remains very low. What diagnosis should be considered?
Recurrent gross hematuria and persistently low C3 are extremely rare in ASPGN. C3 glomerulopathy is a recently characterized disease that includes dense deposit disease (DDD)
and C3 glomerulonephritis (C3GN). Evaluation includes testing for alternative complement regulatory genes, presence of C3 nephritic factor (C3NeF), and a kidney biopsy. The histological
feature is glomerular deposits of C3 or dense deposits in the glomerular basement membrane. Genetic abnormalities occur in the complement alternative pathway (AP). The serum C3 level is often low but C4 is normal. Acquired alternate pathway (AP) dysregulation in DDD and C3GN may be induced by C3 nephritic factor (C3NeF), which is found in 80% of patients with DDD and 45% with C3GN. C3GN may lead to ESRD within 10 years of the diagnosis in 36% to 50% of patients. Recurrences may occur after renal transplantation. Inhibition of complement C3 or C5 is a promising treatment option.

Prasto J, Kaplan BS, Russo P, et al: Streptococcal infection as possible trigger for dense deposit disease (C3 glomerulopathy), Eur J Pediatr 173:767–772, 2014.
Servais A, Noël LH, Frémeaux-Bacchi V, et al: C3 glomerulopathy, Contrib Nephrol 181:185–193, 2013.

64. You are on rounds and a know-it-all nephrology fellow asks you “what are the renal complications of HIV infection?”
The short answer is “any and every renal syndrome or condition.” Human immunodeficiency virus (HIV) nephropathy can result from direct kidney infection with HIV or from the adverse effects of antiretroviral drugs. Patients with HIV disease are also at risk for developing prerenal azotemia due to volume depletion as a result of salt wasting, poor nutrition, nausea,
or vomiting.
When faced with a known clinical problem with an unexpected complication, ask yourself three questions:
• Is the complication related to the disease itself?
• Is it related to the treatment of the disease?
• Is it unrelated to either?
This approach led to the observation that tenofovir was the cause of rickets and renal Fanconi syndrome in a boy with congenital HIV infection.

Wood SM, Shah SS, Steenhoff AP, et al: Tenofovir-associated nephrotoxicity in two HIV-infected adolescent males, AIDS Patient Care STDS 23:1–4, 2009.

HEMATURIA
65. How common is hematuria in children?
Microscopic hematuria (>5 RBCs/high-power field [HPF]) is common (0.5% to 2% of school-age children) and often transient. In 70% to 80% of cases, no etiology is identified.

66. What is the most identifiable cause of microscopic hematuria?
Hypercalciuria, defined as elevated urinary calcium excretion without concomitant hypercalcemia. In areas of the southeastern United States, often called “the stone belt,” this is a common cause of isolated hematuria; nearly one-third of children with microscopic hematuria have hypercalciuria as the cause. It is less common in other parts of the United States. Overall, 3% to 6% of children have idiopathic hypercalciuria.

Srivastava T, Schwaderer A: Diagnosis and management of hypercalciuria in children, Curr Opin Pediatr 21: 214–219, 2009.

67. What distinguishes lower from upper tract bleeding?
As a general rule, brown, tea-colored, or cola-colored urine suggests upper tract bleeding, whereas bright red blood suggests lower tract bleeding (Table 12-6). The darker urine has had more time to become oxidized in the urinary tract. However, exceptions occur. Rapid upper tract bleeding may be red, and a dissolving clot within the bladder may produce brown urine. Establishing the source of microscopic hematuria can be difficult. Glomerular bleeding produces small and dysmorphic RBCs with blebs or burr cells as opposed to the normal-sized RBCs in lower tract bleeding. These changes are best observed with phase-contrast microscopy, which is not readily available in most clinical settings. The presence of significant proteinuria also suggests upper tract (kidney) disease.
The presence of even a single RBC or hemoglobin cast indicates a glomerular (or, rarely, tubular) etiology.

Table 12-6. Glomerular and Nonglomerular Hematuria
GLOMERULAR HEMATURIA NONGLOMERULAR HEMATURIA
History
Burning on micturition Systemic complaints Pain
Trauma Family history
No
Edema, fever, pharyngitis, rash, arthralgia
IgA nephropathy—flank pain No
Deafness in Alport syndrome, renal failure
Urethritis, cystitis
Fever with urinary tract infections.
Calculi—costovertebral pain, radiating pain to groin
Bright red urine
May be positive with calculi
Physical Examination
Hypertension Often present Unlikely
Edema May be present No
Abdominal mass No Wilms tumor, polycystic kidneys
Rash, arthritis Lupus erythematosus, No
Henoch-Sch€onlein
Urinalysis
Color Brown, tea, cola Bright red
Proteinuria Often present No
Dysmorphic red blood Yes No
cells
Red blood cell casts Yes No
Crystals No May be informative in patients with calculi
Kaplan BS, Pradhan M: Helping the pediatrician to interpret urinalysis, Pediatr Ann 42:45–51, 2013.

68. Why does recurrent macroscopic hematuria “gross” us out?
Gross hematuria is problematic because we may not determine a cause and the uncertainty results in parental anxiety and second/third opinions. The most common diagnoses in children with glomerular gross hematuria are IgA nephropathy and Alport syndrome. No cause will be found in the majority of children with glomerular gross hematuria. The most common diagnoses in patients with nonglomerular gross hematuria are hypercalciuria, urethrorrhagia, and hemorrhagic cystitis. Recurrences of gross hematuria are uncommon. In nearly half of the patients with nonglomerular gross hematuria no diagnosis can be established, but their long-term prognosis appeared to be good.

Youn T1, Trachtman H, Gauthier B: Clinical spectrum of gross hematuria in pediatric patients, Clin Pediatr
45:135–141, 2006.

69. If a healthy 10-year-old boy has bright red blood at the end of a previously clear urine stream, what is the likely diagnosis?
Urethrorrhagia. In a preadolescent or early adolescent male, terminal hematuria, which may present with bloodstained underpants, often reflects engorged vessels around the entry of the prostatic duct into the urethra at the veru montanum. Although the etiology is unclear, it is a benign condition associated with hormonal changes at adolescence. It resolves spontaneously in weeks to months and does not require cystoscopy or other investigations.
70. A concerned mother of infant twins brings in a diaper from each with
one having pink urine stains and the other blue urine stains. Which is more worrisome?
Blue diaper syndrome is more worrisome because it is a rare, autosomal recessive inborn error of amino acid metabolism caused primarily by defects in tryptophan intestinal absorption. Increased intestinal bacterial degradation of tryptophan results in increased production and absorption of indican (a protein breakdown product). Increased indicanuria occurs, which on exposure to air
oxidizes to an indigo blue color. The condition can be associated with visual problems, hypercalcemia, and nephrocalcinosis.
Pink diaper syndrome, on the other hand, is a benign condition often misinterpreted as hematuria. The red-brown spotting is caused by normal urate crystals, which turn pink on exposure to air and form a powder (unlike blood) that can be scraped from the diaper (unlike blood).
71. What evaluations should be considered during the evaluation of isolated hematuria?
This is a subject of endless debate. There are two main approaches—one is an unfocused approach and the other is a tailored approach. The thoughtful approach considers hematuria as a symptom of a clinical problem. A careful history and examination must be done.
If the urine is coffee or coke colored and proteinuria and RBC casts are present, consider a glomerulonephritis:
1. Check BUN, serum creatinine, and electrolytes; serologic studies for evidence of a recent streptococcal infection (unless hematuria is recurrent or present for several months); and serum C3.
2. Check antineutrophil cytoplasm antibodies (ANCA) if there is evidence of a vasculitis (fevers, arthralgias, rashes, lung disease).
3. Check antinuclear antibody (ANA) and DNA-binding activity if there are clinical features of systemic lupus erythematosus (SLE). Isolated hematuria almost never occurs in lupus nephritis.
4. Obtain a formal hearing test if there is a family history of Alport disease.
If the urine is bright red and painless:
1. Obtain a renal ultrasound to rule out a Wilms tumor or bladder cancer.
2. Obtain a renal ultrasound to rule out renal calculi (they are not always painful).
3. Obtain a renal ultrasound if there is a history of blunt abdominal trauma to rule out a large renal cyst, dominant polycystic kidney disease, or hydronephrosis.
4. Obtain a hemoglobin electrophoresis if sickle cell trait or disease is suspected.
If the urine is bright red with dysuria:
1. Obtain a renal ultrasound to rule out renal calculi.
2. Obtain a calcium-to-creatinine ratio to rule out hypercalciuria.
3. Obtain a urine culture for a bacterial cause if there are symptoms of a urinary tract infection (UTI).

KEY POINTS: MICROSCOPIC HEMATURIA
1. This is a common finding. Asymptomatic microscopic hematuria is detected in 0.5% to 2% of schoolchildren by the dipstick test.
2. Asymptomatic microscopic hematuria is benign in the majority of individuals.
3. Further evaluation is costly and of no value in the absence of proteinuria, RBC casts, or a family history of renal disease or renal calculi.
4. Hypercalciuria (>4 mg/kg per day) is the most frequent identifiable cause.
5. Patients with significant proteinuria along with hematuria are much more likely to have underlying
pathology.
6. If the dipstick assessment is positive for blood, but microscopic urinalysis is negative for RBCs, consider red dyes from beets or candy, hemolysis (hemoglobinuria) or rhabdomyolysis (myoglobinuria).
7. Take a careful family history of renal disease and of deafness to consider Alport syndrome.
RBC, Red blood cell.

72. What condition should be suspected in a 9-year-old with a history of hearing loss and visual deficits who presents with hematuria?
Alport syndrome, also known as hereditary nephritis should be considered. The cause is one of several genetic mutations that alter a type IV collagen protein essential for glomerular basement membrane function, for integrity of the inner ear organ of Corti, and for maintaining the shape of the lens. The inheritance pattern is mixed: 80% of cases are X-linked,15% are autosomal recessive, and 5% are autosomal dominant.

HYPERTENSION
73. How is hypertension defined in children?
The diagnosis of hypertension is made on the basis of comparison with the normal blood pressures (BP) of healthy children of a similar age, sex, and height. Blood pressure tables are found in the 2004 Fourth Report cited below and on the International Pediatric Hypertension Association (IPHA) website: www.iphapediatrichypertension.org.
Prehypertension: BP readings between the 90th and 95th percentile or >120/80 mm Hg in adolescents.
Hypertension: systolic BP and/or diastolic BP 95th percentile for age, sex, and height (on 3 repeated measurements on different [nonconsecutive] days).

Stage 1 hypertension: BP the 95th percentile and less than the 99th percentile+ 5 mm Hg.
Stage 2 hypertension: BP> 99th percentile+ 5 mm Hg.

National High Blood Pressure Education Program Working Group on High Blood Pressure in Children and Adolescents: The fourth report on the diagnosis, evaluation, and treatment of high blood pressure in children and adolescents, Pediatrics 114:555–576, 2004.
International Pediatric Hypertension Association (IPHA): www.iphapediatrichypertension.org. Accessed on Nov. 25, 2014.

KEY POINTS: HYPERTENSION
1. Common causes of artifactual elevation: The blood pressure cuff is too small, the arm is below the level of the heart, the child is talking and feet are off the ground, or the child is given no time to relax.
2. Common causes of artifactual decrease: The arm is above the level of the heart.
3. Essential (no detectable cause): There is often a strong family history of hypertension.
4. Secondary (detectable lesion) hypertension: The higher the blood pressure and younger the child the more likely there is a secondary cause for the hypertension.
5. Most cases of secondary hypertension in children are caused by renal disease (renal anomalies, renal parenchymal disease) or renovascular disease.

74. Why are repeat visits necessary to diagnose hypertension?
Children and adolescents have very labile BP with prompt response to internal and external stimulae and will have substantial reductions in BP between the first and third visits to a new doctor, in part because of decreased anxiety. Among patients diagnosed as being hypertensive on a first visit to a new physician, there is a mean 15/7 mm Hg fall in the BP by the third visit, with some patients not reaching a stable value until the sixth visit. This does not apply to children or adolescents with repeat BP measurements at a single visit that indicate the presence of stage 2 hypertension. These children need immediate evaluation.

75. How do you determine the optimum cuff size for obtaining a blood pressure? The length of the inflatable bladder inside the cuff (easily palpated) should almost completely encircle the arm and will overestimate the blood pressure if it is too short. Additionally, the height of the cuff should be the largest size that comfortably fits from the axilla to the elbow. A cuff that is too small can produce falsely elevated blood pressure readings.

76. Which Korotkoff sound best represents diastolic blood pressure?
The Korotkoff sounds are produced by the flow of blood as the constricting blood pressure cuff is gradually released. There are five phases of Korotkoff sounds. The first appearance of a clear, tapping sound is called phase I and represents the systolic pressure. As the cuff continues to
be released, soft murmurs can be auscultated; this is phase II. These are followed by louder murmurs during phase III, as the volume of turbulent blood passing through the partially constricted brachial artery increases. The sounds become abruptly muffled in phase IV and disappear in
phase V (usually within 10 mm Hg of phase IV). In studies that compare intravascular blood pressure determinations with auscultatory readings, true diastolic pressure is most closely related to phase V (the disappearance of sound). In some young children, muffled sounds can be heard to “zero” and don’t clearly correlate with diastolic pressure. In these instances, it is best to record both the phase IV (the point at which sounds become muffled) and the phase V readings (e.g., 80/45/0).

77. What is ambulatory blood pressure monitoring (ABPM)?
ABPM is a noninvasive technique for measuring multiple BP readings over a 24-hour period during regular activities and during sleep. It has emerged as an increasingly important tool in the diagnosis and management of children with hypertension. ABPM is performed using an approved ABPM monitor. An appropriately sized BP cuff is placed on the nondominant arm and attached to a small monitor. For 24 hours, BP recordings are taken every 20 minutes while the patient is awake and every 30 minutes while asleep. ABPM is considered satisfactory if there is a minimum of 40 readings during the 24 hours with at least 6 “sleep” readings.

78. What are the advantages and limitations of ABPM?
Advantages: ABPM measurements are made outside of the health-care environment and second multiple parameters of BP can be assessed (mean 24-hour, daytime and nighttime readings, nocturnal dipping, and BP variability). In the general adult population, nocturnal nondipping, nocturnal hypertension, and increased BP variability are predictive of cardiovascular morbidity and mortality. ABPM also permits diagnosis of masked hypertension (normal office BP but
ambulatory BP >95th percentile for sex and height).
Limitations: There is uncertainty about normative BP measures, difficulty in defining
ambulatory hypertension, technical limitations and costs.

Flynn JT, Daniels SR, Hayman LL, et al: Update: ambulatory blood pressure monitoring in children and adolescents. A scientific statement from the American Heart Association, Hypertension 63:1116–1135, 2014.

79. In what settings is ABPM particularly useful?
White-coat hypertension (WCH): WCH is defined as BP levels that are 95th percentile when measured in the office but are completely normal (average BP <90th percentile) outside of the clinical setting. Office measurements often fail to account for this transient, stress-induced elevation of BP. WCH is extremely common in children. Children and adolescents with WCH have increased body mass index (BMI) and a

tendency toward an elevated left ventricular mass index, thereby strengthening the suggestion for clinical ABPM follow-up of WCH.
Masked hypertension: This is the opposite. Patients are truly hypertensive, but the diagnosis is missed in office measurements. The incidence of this phenomenon can be particularly high in children with CKD.

Chaudhuri A: Pediatric ambulatory blood pressure monitoring: diagnosis of hypertension, Pediatr Nephrol
28:995–999, 2013.

KEY POINTS: WAYS TO AVOID MISDIAGNOSING HYPERTENSION
1. Properly sized cuff (age-dependent) with arm supported and kept at heart level
2. Quiet room, quiet patient, seated, feet on the floor or on a stool
3. Repeated measurements over time and the use of averaged values
4. Get rid of the white coat
5. Sit at the child’s level when taking the measurement

80. When should hypertension be treated in the neonate?
As a general rule, hypertension is defined as a blood pressure >90/60 mm Hg in term neonates and >80/45 mm Hg in preterm infants, but strict definitions are unavailable given limited data. A sustained systolic blood pressure of >100 mm Hg in the neonate should be investigated and treated. The most common cause is renovascular disease.

Batisky DL: Neonatal hypertension, Clin Perinatol 41:529–542, 2014.

81. What are the indications for the pharmacologic treatment of hypertension in children and in adolescents?
• Symptomatic hypertension (e.g., headaches, visual disturbances, seizures)
• Stage 2 hypertension
• Stage 1 hypertension (without target-organ damage) not responding to 4 to 6 months of nonpharmacologic therapy (e.g., weight reduction, exercise, decreased salt intake)
• Secondary hypertension
• Hypertensive target-organ damage (left ventricular hypertrophy on ECG)
• Diabetes (types 1 and 2)

National High Blood Pressure Education Program Working Group on High Blood Pressure in Children and Adolescents: the fourth report on the diagnosis, evaluation, and treatment of high blood pressure in children and adolescents, Pediatrics 114:555–576, 2004.

82. During the evaluation of a child with elevated blood pressure, what risk factors should be considered for identification and/or reduction?
Important risk factors for hypertension in children:
• Family history (If one parent has hypertension, the risk is about 25%; if both parents have hypertension, the risk is 45%.)
• Genetic factors including ethnicity (Blacks have twice the incidence of hypertension compared with whites, beginning in adolescence.)
• Obesity
• History of renal disease
• Dietary factors (mainly salt intake)
• Low birth weight
• Since hypertension is a critical risk factor for cardiovascular disease, additional important cardiovascular risk factors should also be assessed. These include diet, serum lipids, tobacco use, and lack of exercise.

83. What historical information suggests a secondary cause of hypertension?
• Known UTI; recurrent abdominal or flank pain with frequency, urgency, dysuria; and secondary enuresis are suggestive of a secondary cause of hypertension.
• Joint pains, rash, fever, edema suggest a vasculitis.
• A complicated neonatal course requiring use of an umbilical artery catheter suggests renal artery stenosis.
• Renal trauma suggests renal artery stenosis.
• Hypertension suggested by drug use (sympathomimetics, anabolic or corticosteroids, NSAIDs, oral contraceptives, illicit drugs) may be responsible for drug-induced hypertension.
• Aberrant course or timing of secondary sexual characteristics or virilization suggests an adrenal disorder.
• Nervousness, personality changes, sweating, flushing suggest a pheochromocytoma or hyperthyroidism.
84. What is the most common cause of renal artery stenosis in children in the United States?
Fibromuscular dysplasia, a nonatherosclerotic, noninflammatory arterial disease of unclear cause is the most common cause. In Asian children, it may be Takayasu arteritis (a vasculitis). This contrasts with adults for whom atherosclerosis is the most common cause. Gold standard for diagnosis is catheter angiography, but other less invasive tests, such as magnetic resonance angiography (MRA), are useful (Fig. 12-3). The vessel narrowing results in afferent arterioles of the kidney sensing a decreased systemic blood pressure due to reduced blood flow. The renin-angiotensin-aldosterone system is activated, which can contribute to hypertension.

Figure 12-3. Renal arteriogram showing renal artery stenosis (arrow) secondary to fibromuscular dysplasia. (From Kaplan BS, Meyers KEC: Pediatric Nephrology and Urology: The Requisites in Pediatrics. Philadelphia, 2004, ELSEVIER Mosby, p 119.)

85. List the features on physical examination that suggest a secondary cause of hypertension
See Table 12-7.

Table 12-7. Physical Findings That Suggest a Secondary Cause of Hypertension
PHYSICAL FINDING POSSIBLE SECONDARY CAUSE
Blood pressure: >140/100 mmHg at any age Multiple secondary causes
Leg< arm blood pressure, decreased or delayed leg pulses Coarctation of the aorta
Adenotonsillar hypertrophy Obstructive sleep apnea
Muscle weakness Hyperaldosteronism
Joint swelling Lupus nephritis, collagen vascular disease
Poor growth Chronic renal disease
Turner syndrome Coarctation of the aorta
>5 café-au-lait spots or neurofibromas Renal artery stenosis, pheochromocytoma
Bruits over large vessels Arteritis
Bruits over mid abdomen Renal artery stenosis
Flank or upper quadrant mass Renal malformation, renal or adrenal tumor
Excess sweating, increased resting heart rate Pheochromocytoma, hyperthyroidism
Excessive virilization or secondary sex characteristics inappropriate for age Adrenal disorder
Edema Renal disease

86. What are the categories of antihypertensive medications used for outpatient management of hypertensive children?
• ACE inhibitors
• Angiotensin receptor blockers (ARB)
• Calcium channel blockers (CCB)
• α-Blockers and β-blockers
• Central α-agonists
• Vasodilators
• Diuretics

Chaturvedi S, Lipszyc DH, Licht C, et al: Pharmacological interventions for hypertension in children. Cochrane Database Syst REV 2:CD008117, 2014.

87. A 12-year-old girl is referred for evaluation of severe hypertension. She has hypernatremia, hypokalemia, and metabolic alkalosis, and plasma renin activity (PRA) on an ACE-inhibitor is not detectable. What is the most likely diagnosis?
Low renin monogenic hypertension. Arterial hypertension in childhood may be due to single gene mutations inherited in an autosomal dominant or recessive fashion. Consider a genetic cause if there are abnormal potassium levels (low or high) in the presence of suppressed renin secretion and metabolic alkalosis or acidosis.

Simonetti GD, Mohaupt MG, Bianchetti MG: Monogenic forms of hypertension, Eur J Pediatr 171:1433–1439, 2012.

88. Why should patients with hypertension and/or those using diuretics avoid true licorice? True licorice contains glycyrrhizic, which indirectly has mineralocorticoid properties (i.e., fluid and sodium retaining, potassium reducing). However, most American licorice contains only licorice flavoring without any such properties. Some chewing tobacco and chewing gum also contains licorice and has been associated with an excessive mineralocorticoid syndrome. Think of this if you are called to evaluate an edematous Boston Red Sox batboy.

de Klerk GJ, Nieuwenhuis MC, Beutler JJ: Hypokalaemia and hypertension associated with use of liquorice flavoured chewing gum, BMJ 314:731–732, 1997.

PROTEINURIA/NEPHROTIC SYNDROME
89. How does the dipstick test for urine protein compare with the sulfosalicylic method? Dipstick assessment relies on the reaction of protein (primarily albumin) with tetrabromophenol blue in a citrate buffer impregnated on the dipstick patch. Mild false-positive reactions can occur (1 + to 2 +) when the patient’s urine is alkaline or when the dipstick is allowed to sit in the urine for too long
and the buffer strength is overcome. The results are reported qualitatively as 1 + to 3 +, which corresponds to a range of 30 to 500 mg/dL.
Sulfosalicylic acid test precipitates protein in the urine and allows for a comparison with a group of previously prepared aqueous standards; it is reported in the same way as those standards. In contrast with the dipstick assessment, all proteins—not just albumin—are precipitated, as are iodinated contrast material and some antibodies. Finding heavy proteinuria by sulfosalicylic acid testing with minimal proteinuria by dipstick testing suggests the presence of large amounts of non-albumin protein. This occurs in adults as the result of multiple myeloma and the excretion of Bence Jones proteins and in children in X-linked Dent disease (a renal tubulopathy).
90. What is microalbuminuria? Microalbuminuria refers to the presence of small (micro) amounts of albumin in urine. It does not refer to small-sized albumin!
91. In what clinical setting is microalbuminuria a particularly important finding?
Type 1 diabetes mellitus. Long-term complications include blindness, kidney damage, cardiovascular disease, and neuropathy. Care for persons with diabetes is focused on early detection and prevention of nephropathy, which occurs in 10% to 20% of diabetics. The earliest evidence of nephropathy is microalbuminuria (MA), defined as the presence of small quantities of albumin in the urine (30 to 300 mg/ 24 hours). Prompt treatment of MA may delay or prevent complications. A first morning sample is recommended, but a random sample is acceptable. MA may be increased with exercise, illness, hypertension, marked hyperglycemia, and diurnal patterns. Therefore, it is advisable to obtain 3 samples over 3 months to avoid false-positive screening results.

Montgomery KA, Ratcliffe SJ, Baluarte HJ, et al: Implementation of a clinical practice guideline for identification of microalbuminuria in the pediatric patient with type 1 diabetes, Nurs Clin North Am 48: 343–352, 2013.

92. What is the best spot method for determining the amount of proteinuria?
Urine protein/creatinine excretion ratio (PCR). A 24-hour urine collection for protein is difficult to obtain in children and is imperfect in adults. Although both the dipstick and the sulfosalicylic methods estimate the concentration of protein in urine, small amounts of protein in very concentrated urine will register as more positive than the same amount of protein in dilute urine. The PCR approximates the total 24-hour urinary protein excretion. On a random sample, a PCR
of < 0.2 to 0.25 reflects normal daily protein excretion, whereas values of >2 suggest the presence of nephrotic-range proteinuria. This test is effective for the diagnosis of the nephrotic
syndrome and for follow-up evaluations in children with prolonged and difficult-to-manage proteinuria. However, the test may overestimate protein excretion in individuals with low muscle mass and therefore lower creatinine excretion. Multiplying the ratio by 0.63 gives an approximation for the 24-hour protein excretion in g/24 hours.

93. An asymptomatic 11-year-old boy is found to have 2 + protein on dipstick testing during a routine checkup. How should he be evaluated?
Assuming that he is healthy and without subtle signs of renal disease (e.g., short stature, pallor, hypertension), and assuming that this is isolated proteinuria without hematuria, always determine whether the proteinuria is orthostatic, intermittent, or persistent.
• Intermittent (transient) proteinuria is entirely benign and does not require any evaluation.
• Persistent proteinuria may or may not be benign. The presence of persistent proteinuria can be determined by rechecking the urine at least 3 times over 2 to 3 weeks. One of these tests should be performed on a first-morning urine specimen (ask the child to void before going to bed the night before).
• Orthostatic proteinuria is determined in the same way. Orthostatic proteinuria is benign. Causes of transient or orthostatic proteinuria include fever, vigorous exercise, dehydration, stress, cold exposure, and seizures.
94. How is the diagnosis of orthostatic proteinuria established?
By definition, individuals with orthostatic proteinuria, who are usually adolescents, have normal rates of protein excretion when recumbent, but have increased excretion rates when upright/ambulant. Although all individuals excrete more protein when standing, some have an exaggerated response and may excrete as much as 1 g/day of protein. The adolescent is instructed to empty the bladder before going to bed and collect a first-morning urine specimen immediately on arising. Protein excretion is then assessed with a urine dipstick or in the laboratory as the PCR. In a concentrated first-morning urine specimen (urine specific gravity
1.018), a trace or negative value by dipstick assessment or sulfosalicylic acid precipitation rules out proteinuria. At any urine specific gravity, a urine PCR of <0.25 is normal. Remember, even individuals with renal disease may have increased protein excretion when standing and lower protein excretion rates when recumbent. The diagnosis of orthostatic proteinuria requires that protein excretion is truly normal when recumbent, elevated when ambulatory, and the individual is otherwise entirely healthy.
95. What additional evaluation should be done for a patient with persistent proteinuria? If the proteinuria is persistent and not orthostatic, the amount of protein excretion must be determined. A timed 24-hour urine collection may be difficult to obtain in children because variable amounts of urine are often lost. The standard definition of proteinuria is the excretion of >4 mg/m2 of protein per hour (or 100 mg every 24 hours for a 30-kg child). In practice, however, the urine PCR is used more frequently
to assess proteinuria. The evaluation of a child with persistent proteinuria includes the same tests required to evaluate glomerulonephritis. Staged investigations include:

Stage 1: Assess BUN, serum creatinine, serum electrolytes, serum albumin, C3 and C4 levels.
Stage 2: Assess ANA, anti-DNA antibodies, and ANCA depending on clinical findings.
Stage 3: Renal imaging studies and renal biopsy may be necessary for diagnosis.
96. What is the natural history of orthostatic proteinuria?
The long-term outcome of children and adolescents is benign. Most agree that the prognosis is excellent, although the etiology remains unclear.
97. What level constitutes significant proteinuria?
Protein excretion of more than 4 mg/m2 per hour on a timed urine collection is abnormal. Children with nephrotic syndrome excrete more than 40 mg/m2 per hour. The upper limit of protein excretion in adults is 150 mg/day, but adolescents may excrete as much as 250 mg/day. A urine protein/creatinine ratio of >0.5 in children <2 years old and of >0.2 in older children is abnormal.
98. In a child with hematuria, can proteinuria be attributed to the protein that is contained in whole blood?
Only in a child with grossly bloody urine. If the urine is normal in color (yellow or clear), any amount of protein above trace is abnormal.
99. Can proteinuria be caused by leukocytes or mucus in the urine? Probably not, although this untested statement is passed on from one generation of physicians to the next. Regardless of whether mucus or leukocytes can yield a positive dipstick test for albumin, it is important to do a spot protein/creatinine ratio if the test is >1 positive.
100. At what serum albumin concentration does edema develop?
Edema starts to manifest when the serum albumin decreases to< 2.5 g/dL. Edema is almost always present at concentrations of <1.8 g/dL unless the child is receiving a diuretic or suffers from the rare

condition of congenital analbuminemia (a very rare condition of low levels of albumin due to impaired synthesis but compensated by increased amounts of other circulating plasma proteins).
101. What is the most common form of nephrotic syndrome in childhood?
Minimal-change nephrotic syndrome (MCNS), previously known as lipoid nephrosis and nil disease, is the most common form. Most patients with MCNS have favorable therapeutic responses and prognoses. Unfortunately, many have frequent relapses, some are steroid dependent, and a minority is steroid resistant. The etiology of MCNS is unknown, but the pathogenesis is related to abnormal
T-lymphocyte function.
102. What is the most important historical factor to consider when assessing a patient for possible MCNS?
The only definitive way to prove MCNS is with a renal biopsy, but this is rarely indicated. Age at presentation is the most important characteristic. Between 75% and 80% of children with nephrotic syndrome have MCNS, and about 80% of those present within the first 8 years of life. It is unusual to manifest before a year of age. Early onset in the first 6 months of life suggests a diagnosis of one of the types of congenital nephrotic syndrome or a secondary cause such as congenital syphilis.
103. What are the typical clinical features and therapeutic responses seen in patients with MCNS?
Edema is generally present, blood pressure is normal to slightly increased, and gross hematuria is absent, but up to one-third may have microscopic hematuria without RBC casts. In the absence of significant intravascular volume depletion, BUN, creatinine, and serum electrolytes are all within normal limits. Serum calcium is low because of hypoalbuminemia. Children who present with these findings should be started on daily prednisone. Up to 90% respond in 1½ to 4 weeks and have steroid-sensitive nephrotic syndrome (SSNS). A response is indicated by a diuresis and a negative or trace dipstick test for protein. If therapy is prolonged for an additional month, another 4% will respond. About 3% of children with biopsy-proven MCNS will be steroid resistant despite 2 months of therapy.
104. What are the indications for furosemide and albumin infusions in patients with nephrotic syndrome?
Indications are severe edema with incapacitating anasarca, cellulitis, skin breakdown, or respiratory embarrassment from pleural effusions. Albumin alone is helpful for a patient with a rising BUN caused by decreased renal perfusion, which is most often seen after vigorous diuretic therapy. Infusion of a 25% albumin solution in a dose of 0.5 to 1 g/kg of albumin over 1 to 2 hours, followed by furosemide (1 to 2 mg/kg), can be used to induce diuresis in a child with nephrotic syndrome who is unresponsive to furosemide alone. This measure is only temporary because the rise in albumin will lead to increased protein excretion, thereby returning the serum level to the previous steady-state value.
105. Name two important complications of MCNS
• Hypercoagulable state, which may result in sagittal sinus, cavernous sinuses, and renal veins thrombosis
• Peritonitis caused by S. pneumoniae or E. coli
106. What are the prognostic factors in MCNS?
The most important prognostic feature in MCNS is a complete response to corticosteroid therapy. However, even a partial response, with a decrease in protein excretion, appears to improve the prognosis. Persistent proteinuria beyond 4 to 6 weeks is a poor prognostic sign and is an indication for cyclophosphamide or tacrolimus treatment and may be an indication for a kidney biopsy.
107. A 5-year-old child presents with puffy eyes and the laboratory features of the nephrotic syndrome, but fails to respond to corticosteroids within 6 weeks. What is the most likely diagnosis? Focal segmental glomerulosclerosis (FSGS). This important type of nephrotic syndrome (accounting for about 20% of cases in children) can progress to ESRD. FSGS is believed to represent a group of clinical-pathologic syndromes that share a common glomerular lesion, which is identified by renal biopsy (Fig. 12-4). A positive biopsy does not confer a disease diagnosis, but represents the beginning of an exploratory process that may lead to identification of a specific etiology and its appropriate treatment. No causes have been found in many cases but increasing numbers of genetic causes are being found, most with autosomal recessive (podocin mutations) and others with dominant modes of inheritance

(ACTN4 mutations). These mutations affect podocyte structure, actin cytoskeleton, calcium signaling, and lysosomal and mitochondrial function. HIV nephropathy and morbid obesity are important causes of secondary FSGS. Patients with a genetic cause of FSGS do not respond to prednisone treatment. Some patient’s may benefit from ACE inhibitors.

Jefferson JA, Shankland SJ: The pathogenesis of focal segmental glomerulosclerosis, ADV Chronic Kidney Dis 21:408–416, 2014.
D’Agati VD, Kaskel FJ, Falk RJ: Focal segmental glomerulosclerosis, N Engl J Med 365:2398–2411, 2011.

Figure 12-4. Focal segmental glomerulosclerosis with partial and segmented sclerosis and lesions of increased extracellular matrix and hyalinosis in light microscopic view with Periodic acid-Schiff staining. (From Johnson RJ, Fehally J, Floege
J, editors: Comprehensive Clinical Nephrology, ed 5. Philadelphia, Saunders, 2015, p 222.)

RENAL FUNCTION ASSESSMENT AND URINALYSIS
108. What is the simplest way to estimate the glomerular filtration rate (eGFR) in the absence of a timed urine collection?
Devising a simple and reliable eGFR is one of the Holy Grails of nephrology. The previous most often used device, the Schwartz formula devised in the mid-1970s, was believed to overestimate GFR. A modified Schwartz formula is now felt to be a more accurate method of estimating GFR.
It requires only a serum creatinine concentration and the height of the child in centimeters. No urine collection is necessary. The modified formula is [0.413 (height in cm/serum creatinine in mg/dL)]. This formula may underestimate eGFR in muscular adolescents and is only validated for CKD stages III through V.

Schwartz GJ, Muñoz A, Schneider MF, et al: New equations to estimate GFR in children with CKD, J Am Soc Nephrol
20:629–637, 2009.

109. How can you be confident that a 24-hour urine collection (for any determination) is complete?
Creatinine is produced continuously and is eliminated only through the kidneys. Therefore, a given amount, determined largely by muscle mass, will be excreted daily, independent of the level of renal function. Thus, the determination of total urine creatinine in a timed sample can give a reasonable estimate of whether the collection approximates that of 24 hours. The guidelines for expected creatinine excretion applicable to children and adolescents are as follows: for males, 15 to 25 mg/kg per day; for females, 10 to 20 mg/kg per day.
110. When should routine urinalyses (UA) be performed in the pediatric age group? There has been some controversy regarding the use of the UA as a routine screening tool. It is a simple, inexpensive, and noninvasive study that is quite sensitive and specific, but the likelihood of this test

uncovering significant, previously undiagnosed renal dysfunction is very low. Because of this, the likelihood of false-positive results is high, leading to unnecessary evaluations. The American Academy of Pediatrics (AAP) in 2007 made the recommendation to discontinue routine urine dipsticks in healthy children as a screen for CKD.

Sekhar DL, Wang L, Hoffenbeck CS, et al: A cost effectiveness analysis of screening urine dipsticks in well-child care, Pediatrics 125:660–663, 2010.
American Academy of Pediatrics: Committee on Practice and Ambulatory Medicine: Recommendations for preventive pediatric health care, Pediatrics 120:1376, 2007.

111. What are the maximal and minimal renal dilutional and concentrating capabilities?
Maximally dilute urine has a specific gravity of 1.001 and an osmolality of 50. Maximally concentrated urine has a specific gravity of about 1.032 and an osmolality of about 1200. Urine that is neither concentrated nor dilute (isosthenuric) has a specific gravity of about 1.010 and a corresponding osmolality of 300. Infants born prematurely do not concentrate urine as effectively.
112. What is the difference between urine specific gravity (SG) and urine osmolality?
Both tests measure the concentration or dilution of the urine, and the relationship between the two is linear and direct, although osmolality is more physiologically correct. Specific GRAVITY is determined by the density (and thus the weight and size) of solute in solution. Osmolality depends on the number of particles (independent of their size) in solution and their effect on changing its freezing point. Therefore, when there are solutes with a relatively large molecular weight (albumin, glucose, contrast material) in the urine, specific gravity will disproportionately increase, and osmolality will be a better indicator of true urine concentration. A urine SG of 1.040 cannot be achieved by the human kidney. Consequently, levels that high in a child with nephrotic syndrome do not represent supernormal concentrating capacity but the effect of heavy proteinuria on the specific gravity.
113. Which urinary crystals are always pathologic?
Cystine crystals. These flat, simple, hexagon-shaped crystals are evidence for the amino acid transport disorder cystinuria (Fig. 12-5). In classic cystinuria, the dibasic amino acids (cystine, ornithine, arginine, and lysine) are affected. The condition would be of little clinical significance except for
the fact that cystine is very insoluble and results in nephrolithiasis.

Figure 12-5. Cystine crystal with hexagonal structure. (From Brown TA, Sonali SJ: USMLE Step 1 Secrets, ed 3. Philadelphia, 2013, ELSEVIER, pp 67–96.)

SURGICAL ISSUES
114. What is the most common cause of urinary tract obstruction in the newborn? Posterior urethral valves. These occur only in boys. The obstruction is frequently associated with high intravesicular pressures, which may damage the renal parenchyma resulting in renal dysplasia if not corrected. Thus, even with prompt recognition and treatment, renal insufficiency may progress.
115. What is the most common renal abnormality detected on antenatal ultrasound?
Hydronephrosis (also known as renal pelvic dilatation) with an incidence between 0.5% and 1% is most common. A renal pelvic diameter 5 mm is typically viewed as a cutoff point with grading (0 to IV) of hydronephrosis based on the degree of dilatation, number of calyces observed and evidence of any parenchymal atrophy. Likelihood of CAKUT increases with the severity of hydronephrosis. A repeat ultrasound is advised at 48 to 72 hours and at 4 to 6 weeks after birth. Based on those results, follow-up studies for vesicoureteral reflux and urology referral may be required. For patients with isolated antenatal hydronephrosis (without evidence of obstruction), routine prophylactic antibiotics are not indicated.
Ninety-eight percent of patients with mild hydronephrosis (renal pelvic diameter <12 mm) resolve, stabilize, or improve at follow-up.

Becker AM: Postnatal evaluation of infants with abnormal antenatal renal sonogram, Curr Opin Pediatr 21:207–213, 2009.

116. What are the possible causes of prenatal hydronephrosis?
• Ureteropelvic junction obstruction (most common)
• Posterior urethral valves
• Vesicoureteral reflux
• Ectopic ureter or ureterocele
• Megaureter (obstructive and nonobstructive)
• Urethral atresia in the prune belly syndrome
117. What is the most common cause of kidney disease of children worldwide? Congenital anomalies of the kidney and urinary tract (CAKUT), which includes obstructive uropathies, ureteropelvic junction obstruction, solitary kidney, renal hypoplasia, and vesicoureteral reflux, present as isolated findings or part of genetic syndromes. CAKUT accounts for up to 40% to 50% of cases of ESRD in children and 7% of ESRD in adults. The genetic mutations that cause CAKUT usually
are sporadic and have been identified in a variety of signaling pathways regulating nephrogenesis. Currently, <10% of affected patients have identified mutations, and most affected patients may have unique genetic diagnoses. There are presently no benefits of offering genetic testing to patients and their relatives with CAKUT.

Copelovitch L, Furth SL: Genetics and urinary tract malformations, Am J Kidney Dis 63:183–185, 2014.

118. Is unilateral renal agenesis (URA) a benign condition?
Yes and no. We used to think so but now we are not so sure. A literature analysis was based on 2684 individuals of whom 63% were males. The incidence of URA was 1 in 2000. Associated CAKUT were identified in 32% of patients, of which VUR was identified in 24% of patients.
Extrarenal anomalies were found in 31% of patients. Hypertension was identified in 16% of patients and 21% of patients had microalbuminuria. Ten percent of patients had a lower GFR (<60 mL/min/1.73 m2).

Westland R, Schreuder MF, Ket JC, van Wijk JA: Unilateral renal agenesis: a systematic review on associated anomalies and renal injury, Nephrol Dial Transplant 28:1844–1855, 2013.

119. Should a child with a single kidney be allowed to play football?
This is a frequently asked question. Most pediatric nephrologists prohibit contact/collision sports participation by individuals with a single kidney, particularly football. The incidence of catastrophic sports-related kidney injury is 0.4 per 1 million children per year from all sports. Cycling was the most common cause of sports-related kidney injury causing >3 times the kidney injuries as football.

In addition, kidney injury from sports is much less common than catastrophic brain, spinal cord, or cardiac injury. Restricting participation of patients with a single, normal kidney from contact/collision sports is unwarranted. Therefore, let the family decide. Keep in mind that a large majority of physicians would ban participation, especially in American football.

Grinsell MM, Showalter S, Gordon KA, Norwood VF: Single kidney and sports participation: perception versus reality,
Pediatrics 118:1019–1027, 2006.

TUBULAR DISORDERS
120. In what settings should renal tubular acidosis (RTA) be considered? Primary RTA is characterized by chronic hyperchloremic metabolic acidosis with an inability to acidify the urine and a normal serum anion gap. Primary RTA is separated into three main types. Signs and symptoms that are common with all forms of RTA are growth failure, polyuria, polydipsia, recurrent dehydration, and vomiting. RTA can also occur secondarily to an acquired renal injury.
121. What are defects in each type of primary RTA?
Type 1 (distal) RTA: Inability of the distal tubule to secrete hydrogen; in the presence of significant acidosis, urine is not maximally acidified (pH <5.5)
Type 2 (proximal) RTA: Decreased ability of the proximal tubule to reabsorb filtered HCO3 at normal
plasma HCO3 concentrations
Type 4 RTA: Acquired or inherited tubular INSENSITIVITY to aldosterone or to an absence of aldosterone
122. What are clinical and laboratory features of the primary RTAs?
See Table 12-8.

Table 12-8. Clinical and Laboratory Manifestations of Various Renal Tubular Acidoses
TYPE 1 (CLASSIC DISTAL)
TYPE 2 (PROXIMAL) TYPE 4 (ALDOSTERONE DEFICIENCY)
Growth failure +++ ++ +++
Serum potassium Normal or low Normal or low High
Nephrocalcinosis Frequent Rare Rare
Low citrate excretion +++ T T
Fractional excretion of filtered HCO3 at normal serum HCO3 levels <5% 5-10% <10%
Daily alkali treatment (mEq/kg) 1-3 5-20 1-3
Daily potassium requirement Decreases with correction Increases with correction
Urine pH >5.5 <5.5 <5.5
Presence of other tubular defects Rare Common Rare

123. How is determining the urine anion gap helpful in the evaluation of metabolic acidosis?
Investigation of any child with a persistent metabolic acidosis must consider some form of RTA in the differential diagnosis. The urinary anion gap is a convenient and accurate screening test for RTA. It is an indirect estimate of urinary ammonium excretion (and thus urinary acid excretion) and is calculated by the following formula after determining urinary electrolyte concentrations:
Urinary anion gap ¼ Na+ + K— — Cl—

If the anion gap is negative, it suggests a large chloride excretion and thus adequate ammonium excretion. The urinary anion gap is negative in hyperchloremic metabolic acidosis as a result of diarrhea, untreated proximal RTA, or prior administration of an acid load. If the anion gap is positive, it suggests an acidification defect, as is seen in patients with distal RTA. Results are not reliable if there are large amounts of unmeasured anions such as ketoacids, penicillin, or salicylates.
124. How is RTA diagnosed with utilizing a urine anion gap in a patient with a hyperchloremic metabolic acidosis and a normal serum anion gap?
See Figure 12.6.

Urine Anion Gap

Negative (Adequate urine NH )

Proximal RTA or GI bicarbonate loss

Urine pH >5.5 and low normal K 

Distal RTA

Positive (Low urine NH )

Urine pH and Plasma K

Urine pH <5.5 and high K 

Type 4 RTA

Figure 12-6. Diagnosis of renal tubular acidosis in patients with hyperchloremic metabolic acidosis and normal serum anion gap. GI, Gastrointestinal; RTA, renal tubular acidosis. (Adapted from Lash JP, Arruda JA: Laboratory EVALUATIon of renal tubular acidosis, Clin Lab Med 13:117-129, 1993.)

125. What is the recommended alkali therapy for the treatment of various forms of RTA? The goals of RTA therapy are to improve growth, correct metabolic bone disease, prevent nephrolithiasis and nephrocalcinosis, and control underlying disease processes.
Alkali therapy (sodium citrate or sodium bicarbonate) is required for all forms of RTA, with the goal of normal plasma HCO3 level. Patients with distal RTA generally require only 2 to 3 mEq of alkali/kg/day. However, infants may also experience some increased urinary bicarbonate wasting and require up to 10 mEq/kg/day. Patients with proximal RTA require large quantities of alkali (5 to 20 mEq/kg/day). For type 4 RTA, patients usually need low-dose alkali therapy (1 to 3 mEq/kg/day) plus a potassium- restricted diet and mineralocorticoid therapy if there is hypoaldosteronism.
126. What is most common cause of the renal Fanconi syndrome? The renal Fanconi syndrome is the manifestation of multiple disorders of transport in the proximal tubule. It is characterized by the abnormal excretion of substances normally reabsorbed by the proximal tubule and for which there is no distal mechanism sufficient to recapture the unabsorbed molecules. Thus, there is abnormal excretion of glucose, phosphate, potassium, amino acids, and bicarbonate. The phosphaturia and hypophosphatemia result in metabolic bone disease. Bicarbonate loss causes metabolic acidosis. Cystinosis, a lysosomal storage disease with abnormal accumulation of the amino acid cystine, is the most common cause. Consider galactosemia, tyrosinemia, and fructose intolerance in any neonate or infant with severe jaundice, acidosis, and glucosuria, which could be a presentation of renal Fanconi syndrome.
127. A 2-year-old girl, generally healthy with height just below the 5th percentile, was noted to have blinking problems and glucose in her urine. Can you connect the two in a single diagnosis?
Patients with cystinosis can have photophobia and renal Fanconi syndrome. Cystine-depleting medical therapy and kidney transplantation has transformed this previously fatal disease into a treatable disorder with a life expectancy of >50 years. Early diagnosis and appropriate therapy are critically important.

Nesterova G1, Gahl WA: Cystinosis: the evolution of a treatable disease, Pediatr Nephrol 28:51–59, 2013.

128. Glucosuria is detected on repeated urine dipstick testing in a 5-year-old boy, but the blood glucose is always normal. What is going on here?
In the absence of clinical symptoms, hypokalemia, metabolic acidosis, or an elevated serum creatinine, the diagnosis is renal glucosuria. This is a benign condition. Because it is commonly familial, genetic abnormalities in renal glucose reabsorption are the likely culprit. Mild glucosuria is typical; heavy glucosuria is rare.
129. What is the clinical presentation of acute interstitial nephritis (AIN)?
AIN is caused by an immune-mediated inflammatory response that initially involves the renal interstitium and tubules, usually sparing the glomeruli and vasculature. AIN has a wide array of clinical presentations that range from isolated tubular disorders (e.g., Fanconi syndrome) to acute renal failure. Additional findings, fever, rash, and arthralgias, may suggest a hypersensitivity reaction.
130. What medications are causes of AIN?
• Antibiotics, especially penicillin analogs, cephalosporins, sulfonamides, and rifampin
• NSAIDs
• Diuretics, especially thiazides and furosemide
131. What laboratory abnormalities are seen in patients with AIN?
The urine sediment is often bland and may be normal aside from a low specific gravity.
• Urinary sediment: May contain RBCs, leukocytes (eosinophils), leukocyte casts
• Urinary protein excretion: Less than 1 g/day; with NSAID use, may be >1 g/day
• Fractional excretion of sodium: Usually >1%
• Proximal tubular defects: Glucosuria, bicarbonaturia, phosphaturia, aminoaciduria
• Distal tubular defects: Hyperkalemia, sodium wasting
• Medullary defects: Sodium wasting, urinary concentrating defects

Meyers CM: Acute interstitial nephritis. In: Greenberg A, editor: Primer on Kidney Diseases. National Kidney Foundation.
San Diego, 1998, Academic Press, p 278.

URINARY TRACT INFECTIONS
132. What two features on the dipstick test are used to evaluate possible UTIs? Nitrite: This test examines urine for the possible presence of nitrites, which can be produced by bacteria possessing the enzyme nitrate reductase, which reduces nitrates to nitrites. False
negatives can occur. Not all urinary pathogens possess the enzyme (e.g., certain Serratia species). The test is more likely to be positive in the setting of a UTI if urine has been present in the bladder for several hours. The test is less effective in infants because of their increased micturition frequency.
Leukocyte esterase: This enzyme, present in white blood cells (WBCs), is typically present when urine is infected. However, since pyuria can be due to other nonbacterial causes and even NSAIDs, the test is less specific.
133. How helpful are dipstick testing and microscopic analysis of urine as screening tests for UTIs?
Sensitivity is the probability that test results will be positive among patients who have UTIs and specificity is the probability that test results will be negative among patients who do not have UTIs. The sensitivity and specificity of the components of the urinalysis individually and in
combination as screening tools for the diagnosis of a UTI are summarized in Table 12-9. Dipstick testing, in particular, is much more effective as a diagnostic tool for UTIs in children >2 years than in younger children.

Mori R, Yonemoto N, Fitzgerald A: Diagnostic performance of urine dipstick testing in children with suspected UTI: a systematic review of relationship with age and comparison with microscopy, Acta Paediatr 99:581–584, 2010.

Table 12-9. Rapid Screening Tests for Urinary Tract Infection in Children: Sensitivity and Specificity
MICROSCOPY SENSITIVITY (%) (RANGE) SPECIFICITY (%) (RANGE)
≤5 WBC/HPF 67 (55, 88) 79 (77, 84)
Any bacteria/HPF 81 (16, 99) 83 (11, 100)
≤5 WBC or bacteria/HPF 99 (97, 100) 65 (67, 74)
Dipstick
Any LE 83 (64, 89) 84 (71, 95)
Any nitrite only 50 (16, 72) 98 (95, 100)
Any nitrite or LE 88 (71, 100) 93 (76, 98)
LE leukocyte esterase; HPF high-power field; WBC white blood cell.
Data from Christensen AM, Shaw K: Urinary tract infection in childhood. In: Kaplan BS, Meyers KEC, editors: Pediatric Nephrology and Urology: The Requisites in Pediatrics. Philadelphia, 2004, ELSEVIER/MOSBy, p 320.

134. Can the diagnosis of UTI be made on the basis of urinalysis alone?
No. A urine culture is the only accurate means of diagnosing a UTI. Urinalysis is valuable for selecting individuals for the prompt initiation of treatment while awaiting results of the urine culture. In older children (in whom UTI symptoms are more reliable indicators of infection), a negative nitrite test, a negative leukocyte esterase test, and the absence of UTI symptoms are highly correlated with the absence of infection. However, babies require a culture to exclude UTI.

KEY POINTS: URINARY TRACT INFECTION
1. Escherichia coli bacteria cause 90% of cases.
2. Antibiotic sensitivity testing is important because of the increasing incidence of ampicillin-resistant
E. coli.
3. Infections may be caused by bacteria ascending from the urethral area.
4. Clean bagged specimens are unreliable for diagnosis because of their high contamination rate.
5. Uncircumcised male infants have a 10-fold greater risk for infection than circumcised male infants.

135. What bacterial counts constitute a positive urine culture?
• Suprapubic aspiration: At least 100 colony-forming units (CFU)/mL
• Catheterization: At least 10,000 CFU/mL
• Midstream clean catch: At least 100,000 CFU/mL of a single organism; 10,000 to 100,000 CFU/mL is suspicious for a UTI and requires reculturing; less than 10,000 CFU/mL usually indicates contamination
• Urine bag: May be helpful if negative, but even when counts are 100,000 CFU/mL, there is a 40% to 85% false-positive result.
136. A child has painful, frequent urination and a culture revealed a UTI, but the original urinalysis had a negative nitrite study. What is the most likely reason?
Members of the gram-negative, rod-shaped Enterobacteriaceae family can reduce dietary
nitrate to nitrite. However, the bacteria need hours for this conversion to occur. A first-morning void is more likely to be positive compared with the urinalysis of a child who has been urinating frequently with insufficient time to incubate in the bladder. A false-negative result is common with the nitrite test.

Patel HP: The abnormal urinalysis, Pediatr Clin North Am 53:325–337, 2006.

137. What factors can cause a low colony count despite a definite urinary infection?
• High urine volume
• Recent antimicrobial therapy
• Fastidious and slow-growing organisms (enterococci, Staphylococcus saprophyticus)

• Low urine pH(<5.0) and Specific gravity (<1.003)
• Bacteriostatic agents in the urine
• Complete obstruction of a ureter
• Chronic or indolent infection
• Use of inappropriate culture techniques

Bock GH: Urinary tract infections. In: Hoekelman RA, Adam HM, Nelson HM, et al: editors: Primary Pediatric Care, ed 4. St. Louis, 2001, Mosby, p 1896.

138. Why should urine specimens be refrigerated if they cannot be immediately processed?
Storage of urine specimens at room temperature is the most common causes of false-positive results. At room temperature, enteric organisms in specimens have a growth-doubling time of 12.5 minutes, and thus colony counts become an unreliable guide. If a urine specimen cannot be processed within
15 minutes, it should be refrigerated at less than 4°C to stop in VITRO bacterial replication.
139. What are the common presenting signs and symptoms of a UTI in an infant? The presenting findings are nonspecific and include fever, vomiting, diarrhea, irritability, hyperbilirubinemia, and poor feeding. These same findings are often seen in infants without UTIs, underscoring the importance of urine cultures in febrile infants.
140. How common are UTIs in young febrile infants?
In infants and toddlers between 2 and 24 months with unexplained fever (>38.3°C) the prevalence is about 7%, but it ranges between 2% and 9% depending on age and sex. The younger the child, the more
likely the presence of a UTI. Girls have twice as many infections (or more) as circumcised boys. White female infants are twice as likely to have a UTI as black infants. In the first 3 months of life, uncircumcised males with fever have a 10-fold increased risk compared with circumcised boys. In infants younger than 2 months, 7.5% are likely to have UTIs, with boys having more than girls. Needless to say, the possibility of a UTI should always be considered in younger infants, particularly those without an identifiable source of infection, because UTIs now constitute the most likely source of an occult bacterial infection by a wide margin.

Greenhow TL, Hung YY, Herz AM, et al: The changing epidemiology of serious bacterial infections in young infants, Pediatr Infect Dis J 33:595–599, 2014.
Shaikh N, Morone ME, Bost JE, et al: Prevalence of urinary tract infection in childhood: a meta-analysis, Pediatr Infect Dis J 27:302–308, 2008.

141. What pathogens are associated with UTIs in children? Between 80% and 90% of initial UTIs are caused by E. coli. Other organisms include Proteus mirabilis, Klebsiella pneumoniae, Pseudomonas, Enterobacter, and some Staphylococcus species.
142. How is cystitis distinguished clinically from pyelonephritis?
This can be difficult. Pyelonephritis tends to have more constitutional symptoms, such as fever, rigors, flank pain, and back pain, whereas cystitis has more bladder symptoms, such as enuresis, dysuria, frequency, and urgency. The presence of WBC casts or impaired urinary-concentrating ability is
more indicative of pyelonephritis. Patients with pyelonephritis tend to have higher sedimentation rates, C-reactive protein levels, and serum procalcitonin levels, but these results can also be seen in some patients with cystitis. Renal dimercaptosuccinic acid (DMSA) scintigraphy may be useful for identifying acute pyelonephritis. However, for most children, the treatments for cystitis and of pyelonephritis are the same.
143. What is the diagnostic approach for a possible UTI for a female infant 3 to 24 months of age with no known urinary tract abnormalities?
One algorithmic approach uses risk factors and likelihood ratios (a number <1 is less likely, >1 more likely) and urinalysis results to categorize the probability of UTI (Fig. 12-7). Additional diagnostic algorithms are also available at the JAMA reference for febrile males ages 3 to 24 months and for verbal children> 24 months with urinary or abdominal symptoms.
Shaikh N, Morone NE, Lopez J, et al: Does this child have a urinary tract infection? JAMA 298:2895–2904, 2007.

Febrile female infant aged 3 to 24 months with no known urinary tract abnormalities

Age <12 months Probability of UTI ~7%

UTI risk factors History of UTI Temperature >39°C
Fever without apparent source III appearance
Suprapubic tenderness Fever >24 h
Nonblack race

Age <12 months Probability of UTI ~2%

>1 UTI risk factors? >1 UTI risk factors?

Obtain urinalysis
and urine culture Probability of
UTI ~10% to 25%

Clinical followup in
24 hours to reassess risk of UTI Probability of
UTI <2%

Obtain urinalysis
and urine culture Probability of UTI ~3% to 8%

Urine dipstick nitrite and leukocyte esterase negative (LR = 0.2) Probability of UTI
~2% to 6%

Urine dipstick nitrite and leukocyte esterase positive (LR = 28) Probability of UTI
~75% to 90%

Urine dipstick nitrite and leukocyte esterase
negative (LR = 0.2) Probability of UTI
<2%

Urine dipstick nitrite and leukocyte esterase positive (LR = 28) Probability of UTI
~46% to 71%

Urine dipstick nitrite or
leukocyte esterase positive (LR = 6) Probability of UTI
~40% to 66%

Urine dipstick nitrite or
leukocyte esterase positive (LR = 6) Probability of UTI
~15% to 34%

Figure 12-7. Diagnostic algorithm for febrile female infants 3 to 24 months of age suspected of having a urinary tract infection. LR, Likelihood ratio; UTI, urinary tract infection. (From Shaikh N, Morone NE, Lopez J: Does this child HAVE a urinary tract infection? JAMA 298:2902, 2007.)

144. Which patients with UTIs require hospitalization and parenteral antibiotics?
• Any patient who is toxic, dehydrated, or unable to tolerate oral antibiotics.
• A patient with an underlying urinary tract abnormality in which pyelonephritis is suspected.
• Many centers will hospitalize any infant <2 months because of a concern of an increased risk for urosepsis or other serious concomitant infections. However, recent studies indicate that low-risk infants (i.e., not clinically ill, no significant past medical history, normal WBC
indices) may be at very low risk for bacteremia or clinical decompensation and might be managed with brief hospitalization or as outpatients. That debate is ongoing.

Schnadower D, Kuppermann N, Macias CG, et al: Outpatient management of young febrile infants with urinary tract infections, Pediatr Emerg Care 30:591–597, 2014.
Schnadower D, Kuppermann N, Macias CG, et al: Febrile infants with urinary tract infections at very low risk for adverse events and bacteremia, Pediatrics 126: 1074–1083, 2010.

145. Should all pediatric patients with clinical pyelonephritis be hospitalized?
The short- and long-term outcomes of patients (even as young as 2 months old) with uncomplicated pyelonephritis are the same whether they are treated initially with intravenous antibiotics or with oral, third-generation cephalosporins. A decision about outpatient therapy mandates the ability to tolerate oral antibiotics with no concerns regarding compliance and careful and reliable follow-up.

Strohmeier Y, Hodson EM, Willis NS, et al: Antibiotics for acute pyelonephritis in children, Cochrane Database Syst REV 7:CD003772, 2014.

146. What is the expected resolution of fever after a child is started on an antibiotic for a UTI?
In one study of 128 infants younger than 60 days with UTI treated with parenteral antibiotics, 85% became afebrile within 24 hours. Only 4% were febrile after 48 hours. In another study of 364 patients 1 week to 18 years of age, 32% had fever beyond 48 hours. Older age is a risk factor for protracted fever.

Dayan RS, Hanson E, Bennett JE, et al: Clinical course of urinary tract infections in infants younger than 60 days of age, Pediatr Emerg Care 20:85–88, 2004.
Currie ML, Mitz L, Raasch CS, et al: Follow-up urine cultures and fever in children with urinary tract infection, Arch Pediatr Adolesc Med 157:1237–1240, 2003.

147. What is the duration of antibiotic therapy for a UTI?
Data to support a precise duration are insufficient. Standard duration of therapy for cystitis/lower UTIs varies from 7 to 14 days (oral or combined oral plus parenteral). Some experts lean toward 14 days of treatment for pyelonephritis. When IV antibiotics are given, a short IV course (2 to
4 days) followed by oral antibiotics is as effective as a longer course (7 to 10 days) of IV therapy. If the patient has not clinically improved within 2 to 3 days of starting therapy, the urine culture should be repeated and antibiotics adjusted, if indicated. Short-course (2- to 4-day) therapy
compared with standard-duration (7- to 14-day) therapy for lower UTIs have shown clinical equivalency in some studies. Single-day or single-dose therapy is less effective and is not recommended.

Strohmeier Y, Hodson EM, Willis NS, et al: Antibiotics for acute pyelonephritis in children, Cochrane Database Syst REV 7: CD003772, 2014.
Fitzgerald A, Mori R, Lakhanpaul M, et al: Antibiotics for treating lower urinary tract infection in children, Cochrane Database Syst REV 8:CD006857, 2012.

148. Are repeat cultures required at the end of therapy for a patient without symptoms? Although done commonly in the past as a “test of cure,” follow-up urine cultures for a clinically improving patient >2 months of age are not indicated because the yield is extremely low (<.0.5%).

Oreskovic NM, Sembrano EU: Repeat urine cultures in children who are admitted with urinary tract infections,
Pediatrics 119:e325–e329, 2007.

149. In what patients are prophylactic antibiotics indicated for UTIs?
This is controversial. Prophylaxis for recurrent UTI may actually increase the likelihood of infections. The recommendations are watchful waiting and rapid assessment when clinical concerns arise. It is also unclear whether preventing recurrent UTIs will prevent renal scarring. Consequently, prophylaxis for recurrent UTI in children with normal urinary tract anatomy is debatable, particularly for younger infants.
Prophylaxis is generally indicated:
• In infants or children with their first UTI, who have finished their first course of antibiotic therapy and are awaiting the completion of studies (e.g., renal ultrasound).
• In patients with known urologic abnormalities that place them at high risk for recurrent UTIs (e.g., severe voiding disorders, high-grade VUR); however, the value of antibiotics in these situations is also questioned.

Dai B, Liu Y, Jia J, et al: Long-term antibiotics for the prevention of recurrent urinary tract infections in children: a systematic review and meta-analysis, Arch Dis Child 95:499–508, 2010.
Conway PH, Cnaan A, Zaoutis T, et al: Recurrent urinary tract infection in children: risk factors and association with prophylactic antimicrobials, JAMA 298:179, 2007.

150. Is cranberry juice helpful in the management of recurrent UTIs in children?
The use of cranberry juice as a urine-acidifying agent and treatment for UTI has been popular for adults since the 1920s and was used in the 1800s for disorders of the bladder. Studies of adults have shown it to be helpful for diminishing the frequency of bacteriuria, possibly because of its antiadhesive properties against E. coli. Results in pediatric studies are mixed, but more highly concentrated juice may have some limited value in recurrent UTI in children with no urologic abnormalities.

Afshar F, Stothers L, Scott H, et al: Cranberry juice for the prevention of pediatric urinary tract infection: a randomized controlled trial, J Urol 188:1584–1587, 2012.

151. Do patients with an initial UTI require imaging studies? Approaches are controversial because of uncertainties regarding any causal relationship between UTIs, VUR, and renal scarring. In 1999, AAP guidelines recommended renal ultrasonography and voiding cystourethrogram (VCUG) in children< 2 years with a UTI to search for anomalies or the presence of vesicoureteral reflux. In 2011, these AAP guidelines were revised to recommend a follow-up ultrasound
in all cases, but no VCUG routinely after the first UTI. A VCUG was recommended only if ultrasonography revealed hydronephrosis, scarring, or other findings that would suggest either high-grade VUR or obstructive uropathy. VCUG was also recommended in atypical or complex circumstances and for recurrent UTIs. The value of these studies, particularly their role in preventing long-term renal sequelae, continues under reexamination. Since the revised guidelines, clinical trends do indicate a more limited use of imaging studies with emphasis on higher-risk patients and a reduction in the use of the VCUG. Guidelines in the United Kingdom rely on ultrasound and radionuclide studies rather than the VCUG.

Coulthard MG, Lambert HJ, et al: Guidelines to identify abnormalities after childhood urinary tract infections: a prospective audit, Arch Dis Child 99:448–451, 2014.
Roberts KB, Subcommittee on Urinary Tract Infection, Steering Committee on Quality Improvement and Management: Urinary tract infection: clinical practice guideline for the diagnosis and management of the initial UTI in febrile infants and children 2 to 24 months, Pediatrics 128: 595–610, 2011.
Friedman AL: Acute UTI: What you need to know, Contemp Pediatr 25:68–76, 2008.
DeMuri GP, Wald ER: Imaging and antimicrobial prophylaxis following the diagnosis of urinary tract infection in children,
Pediatr Infect Dis J 27:553–554, 2008.

152. What imaging studies can be used for patients with UTIs who warrant evaluation?
• Renal ultrasound, to screen for urinary tract obstruction or other structural genitourinary abnormalities
• VCUG or radionuclide cystogram, to evaluate for vesicoureteral reflux (the most common abnormality found in children with UTIs)
• Renal cortical DMSA or MAG-3 (99mTc-mercaptoacetyltriglycine) scanning, recommended by some authorities to determine whether there is evidence of acute pyelonephritis or permanent renal scarring (Fig.12-8)

Figure 12-8. DMSA renal scan showing renal scarring (arrow) from pyelonephritis. Lt, left; Rt, right. (From Kaplan BS, Meyers KEC: Pediatric Nephrology and Urology: The Requisites in Pediatrics. Philadelphia, 2004, ELSEVIER Mosby, p 119.)

UROGENITAL ISSUES
153. What are risks associated with circumcision? Rare complications are bleeding and infection. With poor technique, injury or amputation of the glans can occur. Meatal stenosis as a consequence of meatal ulceration is another potential complication.
154. Is circumcision now medically indicated? We have watched the AAP’s position statements on circumcision transform over the past 40 years. In the 1970s, circumcision was viewed as more of a negative. In 1999, the stance was modified to one of neutrality. In the 2012 policy statement, the positive benefits of circumcision were emphasized. Benefits have been found to exceed risks by at least 100 to 1 and that over the course of a lifetime, half of uncircumcised males will require treatment for a medical condition associated with retention of the foreskin. These benefits included protection against UTIs, sexually transmitted infections (particularly HIV infections), balanitis, and phimosis, and a lower incidence of penile cancer. No study has identified any adverse effect on sexual function or pleasure.
Although medical benefits can be emphasized, the decision at present still rests primarily on nonmedical family and cultural considerations.
As of 2013, CDC data indicate that the circumcision rate in the United States is about 80%. Clear racial differences were documented with rates of 91% among whites, 76% among blacks and 44% among Hispanics.

Morris BJ, Bailis SA, Wiswell TE: Circumcision rates in the United States: Rising or falling? What effect might the new affirmative pediatric policy statement have? Mayo Clin Proc 89:677–686, 2014.
American Academy of Pediatrics Task Force on Circumcision: Circumcision policy statement, Pediatrics 130: e756–e785, 2012.

155. What distinguishes phimosis and paraphimosis?
Phimosis is a narrowing of the distal foreskin, which prevents its retraction over the glans of the penis. In newborns, retraction is difficult because of normal adhesions that gradually self-resolve. Chronic inflammation or scarring can cause true phimosis with persistent narrowing and may require circumcision.
Paraphimosis is incarceration of a retracted foreskin behind the glans. It occurs when the retracted foreskin is not repositioned. Progressive edema results, which, if uncorrected, can lead to ischemic breakdown. Local anesthesia, ice, and manual reduction usually correct the problem, but if these are unsuccessful, surgical reduction is necessary.

Huang CJ: Problems of the foreskin and glans penis, Clin Pediatr Emerg Med 10:56–59, 2009.
156. What is hypospadias?
Hypospadias is a congenital defect in which the urethral opening is displaced to the underside of the penis. It results from the failure or delay of the midline fusion of the urethral folds. It is often associated with a ventral band of fibrous tissue (chordee) that causes ventral curvature of the penis, especially with an erection. Incidence is 1 to 2 per 1000 live births. When assessing hypospadias, it is useful to describe where the urethral meatus appears (i.e., glandular, distal shaft, proximal shaft, or perineal) and also the degree and location of chordee. The treatment of hypospadias is surgical repair, usually as a one-step procedure. With the advent of microsurgical techniques, the optimal time for repair appears to be 6 to 12 months of age.
157. How common are undescended testicles at birth?
The answer is very much dependent on gestational age. About 3% of term male infants are affected, but that increases to up to one-third of premature infants. The more premature the infant, the higher the likelihood of an undescended testicle.
158. When should undescended testicles be repaired?
The optimal time for surgery on an undescended testicle is 12 months of age or shortly thereafter.
Traditional teaching is that the majority of newborn cryptorchidism resolves without intervention
with 75% percent of full-term infants and 90% of preterm cryptorchid newborns having full testicular descent by the age of 9 months. Some newer studies suggest the rate of spontaneous descent may be lower. Spontaneous descent after 9 months is unlikely. The AAP recommends surgery around 1 year of age to prevent testicular degeneration and to decrease the risk of testicular cancer. During the

second year of life, ultrastructural changes in the seminiferous tubules of the undescended testes begin to appear, but these may be halted by orchiopexy.

Yiee JH, Saigal DS, Lai J, et al: Timing of orchiopexy in the United States: a quality-of-care indicator, Urology
80:1121–1126, 2012.
Wenzler DL, Bloom DA, Park JM: What is the rate of spontaneous testicular descent infants with cryptorchidism? J Urol
171:849–851, 2004.
159. How do you treat labial adhesions? Labial adhesions are a relatively common gynecologic finding in girls between 4 months 6 years of age. They may be complete or partial and are thought to result from local inflammation in a low-estrogen setting with resulting skin agglutination. Treatment consists of eliminating the underlying inflammation (if caused by an infection), sitz baths twice daily, maintenance of good perineal hygiene, and topical application of a 1% conjugated estrogen cream over the entire adhesion at bedtime for 3 weeks. The use of estrogen has an 80% to 90% cure rate and may be followed by the application of a petroleum jelly for 1 to 2 months nightly. It should be noted that the natural history of untreated asymptomatic labial adhesions is self-resolution: 50% resolve within 6 months, and nearly 100% resolve by 18 months. Surgical correction is almost never required.

Leung AKC, Robson WLM, Kao CP, et al: Treatment of labial fusion with topical estrogen therapy, Clin Pediatr 44:245–247, 2005.

UROLITHIASIS
160. Why are kidney stones increasing in frequency in children in the United States? There has been a 5-fold increase over the past 2 decades. A leading theory is that increased salt intake (consumption of salty snacks and processed foods) and insufficient fluid intake have led to increased urinary calcium and oxalate concentrations and stone formation. The increasing obesity is also paralleling increasing urolithiasis in children.

Copelovitch L: Urolithiasis in children, Pediatr Clin North Am 59: 881–896, 2012.

161. What are the clinical findings in pediatric urolithiasis?
Patients present most commonly with flank pain, usually unilateral, with nausea and vomiting. Although hematuria (>2 RBCs/HPF) is common, up to 15% may have not have detectable hematuria. In about one- third of cases, there is a family history of urolithiasis. Fever, dysuria, and costovertebral angle tenderness lower the likelihood of stones and make infection more likely, although both may occur together.

Persaud AC, Stevenson MD, McMahon DR, Christopher NC: Pediatric urolithiasis: clinical predictors in the emergency department, Pediatrics 124:888–894, 2009.

162. What is the composition of kidney stones in children?
Calcium (58%), struvite (25%), cysteine (6%), uric acid, urate (9%), others (2%)
163. What is the most common cause of pediatric urinary calculi?
Idiopathic hypercalciuria is the most common cause of pediatric urinary calculi. Other causes include:
• Hypercalcemia
• Hypocitraturia
• Hyperoxaluria
• Cystinuria
• Renal tubular dysfunction (usually Type 2 dRTA)
• Endocrine (hypothyroidism, adrenocorticoid excess, hyperparathyroidism)
• Bone metabolism disorders (immobilization, rickets, malignancies, juvenile rheumatoid arthritis)
• Drugs (loop diuretics, excess vitamin D, corticosteroids)
• UTI
• Polycystic kidneys (dominant and recessive)
A comprehensive metabolic evaluation is indicated in all children with calculi because
of the high risk of recurrences in children with idiopathic hypercalciuria and hypocitraturia and

the importance of excluding rare but treatable conditions such as primary hyperoxaluria and cystinuria.
Copelovitch L: Urolithiasis in children, Pediatr Clin North Am 59: 881–896, 2012.
Spivacow FR, Negri AL, del Valle EE, et al: Metabolic risk factors in children with kidney stone disease, Pediatr Nephrol
23:1129–1133, 2008.
164. How is hypercalciuria defined in pediatrics?
The strict definition of hypercalciuria in a child is >4 mg of urinary calcium/kg/24 hr on a normal, unrestricted diet for the child’s age. Random urine collections are used to screen for hypercalciuria. The urine calcium-to-creatinine ratio varies with age. Morning nonfasting urine ratios that exceed the
following criteria correlate with quantitative hypercalciuria:
Age (years) Ratio
>7 >0.24
5-7 >0.30
3-5 >0.41
1-2 >0.56
<1 >0.81
165. What are the appropriate laboratory studies for the initial evaluation of children with renal stones?
• Serum electrolytes, calcium, phosphorus, and creatinine
• 24-hour urine collection for sodium, calcium, creatinine, urate, citrate, uric acid, oxalate, and cystine
• Urine pH (by meter), urinalysis, and urine culture (if indicated)
• Stone analysis on an available stone
• Serum parathyroid hormone and vitamin D in patients with hypercalciuria, hypercalcemia, or hypophosphatemia
Nicoletta JA, Lande MB: Medical evaluation and treatment of urolithiasis, Pediatr Clin North Am 53:479–491, 2006.
166. When is lithotripsy or surgery indicated for children with kidney stones? Most stones up to 5 mm will pass spontaneously. Shock wave lithotripsy (SWL) is useful for children with pelvic or bladder stones that are radiopaque in whom fluoroscopy can be used to focus the shock waves. In general, SWL has a success rate of less than 50% with stones larger than 2 cm. Surgery (percutaneous nephrolithotomy) is reserved for stones causing urinary tract obstruction in children and for staghorn calculi in older patients. Cystine stones are difficult to fragment by SWL. Distal urethral stones are removed by ureteroscopy.
Mandeville JA, Nelson CP: Pediatric urolithiasis, Curr Opin Urol 119:419–423, 2009.

VESICOURETERAL REFLUX
167. How is vesicoureteral reflux (VUR) graded? VUR, the retrograde flow of urine from the bladder into the upper urinary tract, is typically divided into five grades, as shown in Fig. 12-9.

I II III IV V
Figure 12-9. The five grades of vesicoureteral reflux.

168. What is the natural history of VUR? The likelihood that reflux will resolve spontaneously is influenced by the severity of the reflux at the time of the initial diagnosis. About 80% to 90% of patients with grade I to II reflux, 70% (<age 2 years) and with grade III reflux, and 60%% with unilateral grade IV reflux will experience spontaneous resolution within 5 years. Spontaneous resolution of grade V reflux is uncommon. The chances for
resolution are better in younger children and those with unilateral—rather than bilateral—reflux, especially for the higher grades of reflux.

Elder JS, Peters CA, Arant BS Jr, et al: Pediatric Vesicoureteral Reflux Guidelines Panel summary report on the management of primary vesicoureteral reflux in children, J Urol 157:1846–1851, 1997.

169. How is VUR managed: medically or surgically? This is controversial. There are significant institutional differences given differing opinions on the role of VUR as a predisposing factor to acute pyelonephritis, renal scarring, and CKD, as well as the value of prophylactic antibiotics.
Grades I to II: These grades are usually managed medically, usually without prophylactic antibiotics, and given a higher likelihood of spontaneous resolution and a lower likelihood of significant sequelae. Grades III to IV: If followed expectantly, these types of reflux will resolve slowly, at a rate of about 10% per year. Surgical intervention at the ureterovesicular junction will result in the elimination of this
degree of reflux in the vast majority of patients, but randomized studies have not shown any significant difference in long-term renal outcome (e.g., renal scarring, hypertension, reduced function) when comparing medical versus surgical treatment. Prophylactic antibiotics are commonly used for these grades.
Grade V: Surgical intervention is typically indicated, particularly if the patient is older (>6 years) and significant renal scarring has been noted.
Martin AD, Iqbal MW, Sprague MD, et al: Most infants with dilating vesicoureteral reflux can be treated nonoperatively, J Urol 191(5 Suppl):1620–1626, 2014.
Peters CA, Skoog CJ, Arant BS Jr, et al: Summary of the American Urological Association guidelines on management of primary vesicoureteral reflux in children, J Urol 184:1134–1144, 2010.

170. Are antibiotics effective to prevent recurrent UTIs in children with reflux? The effectiveness of antibiotics in the setting of significant reflux has been controversial and the results mixed. A 2014 well-designed Randomized Intervention for Children with Vesicoureteral Reflux (RIVUR) trial has found that treatment of children with VUR (grades I to IV) with a prophylactic antibiotic (trimethoprim-sulfamethoxazole), compared with placebo, was associated with a substantially reduced risk of recurrent UTIs but not of renal scarring for children treated following a first or second UTI. The authors questioned whether this decrease in recurrences in the setting of reflux might spur a reevaluation of the AAP’s recommendation to not routinely do a reflux imaging study after a first UTI.

RIVUR Trial Investigators, Hoberman A, Greenfield SP, et al: Antimicrobial prophylaxis for children with vesicoureteral reflux,
N Engl J Med 370:2367–2376, 2014.

171. Should asymptomatic siblings of a patient with VUR have urologic imaging done as a screen for reflux? Some studies have demonstrated reflux in 33% to 45% of siblings of patients with reflux. Among identical twins, the rate is 80%. Typically, the reflux is milder (grades I to II) with only 2% of siblings in published studies having reflux of grades IV or V. Although the incidence of reflux is higher in siblings, data are still lacking that the screening and treatment of asymptomatic siblings decreases renal scarring.

Routh JC, Grant FD, Kokorowski P, et al: Costs and consequences of universal sibling screening for vesicoureteral reflux: a decision analysis, Pediatrics 126:865–871, 2010.

Acknowledgment
The editors gratefully acknowledge contributions by Drs. Michael Norman, Thomas Kennedy, James Prebis, and Stephen J. Wassner that were retained from the first three editions of Pediatric Secrets.

BONUS QUESTIONS
172. Why are some children with congestive heart failure (CHF) hyponatremic while others are not?
Differences in serum sodium are in large measure dependent on whether CHF is compensated or
uncompensated. Hyponatremia is more likely in the latter.
In the uncompensated state, there is decreased cardiac output with arterial underfilling. This results in activation of volume receptors in the atria, great vessels (carotid sinus), and other sites, leading to sympathetic nervous system stimulation and activation of the renin-angiotensin-aldosterone system. This increases renal sodium reabsorption; the decreased volume stimulates antidiuretic hormone (ADH) release. ADH reduces the kidneys ability to excrete ingested or infused water, and water retention produces a dilutional hyponatremia. The resulting sodium and water retention improves cardiac performance and the position on the Starling curve.
This leads to a new steady state in which the above mechanisms are no longer activated and normal water excretion can reestablish a normal serum sodium concentration. When no further compensation is possible, dilutional hyponatremia will persist in association with total body sodium and water overload.

173. What treatment is available for NDI? Desmopressin is not an option because patients with NDI do not respond to AVP (unlike central diabetes insipidus). Therefore, one must ensure that water is always available to avoid dehydration. Advise parents that these children can become dehydrated quickly during an episode of gastroenteritis, and require more rapid medical intervention with intravenous fluids than an unaffected child. Decreasing the solute load for urinary excretion by lowering salt and protein intake will help decrease urine volume. Add hydrochlorothiazide if polyuria and polydipsia are intolerable. A potassium-sparing diuretic (amiloride) may be started if hydrochlorothiazide produces hypokalemia. A cyclooxygenase inhibitor (indomethacin) can also efficiently decrease the amount of polyuria.
174. Which children with acute kidney injury are at the greatest risk for hyperkalemia? The two most important determinants of potassium excretion are the rate of urine flow and aldosterone. Acute kidney injury can be oliguric, anuric, or nonoliguric. Patients with oliguria and especially anuria are at the greatest risk for hyperkalemia because of reduced GFR, low urine flow rate, metabolic acidosis, and increased catabolism.
175. In a child with HUS, does the presence of a low C3 level have any special significance? Yes. It signifies that the patient likely has aHUS. These children often have a defect in one of the alternate pathway factors that inhibit complement activation of C3; these factors normally prevent the uncontrolled activation of terminal complement components C5 to C9 (the membrane attack complex). Mutations can occur in complement factor H, complement factor I, and membrane cofactor protein (MCP).
Treatment with infusions of fresh-frozen plasma and plasma exchanges have been replaced
by eculizumab, which is now the treatment of choice. Unfortunately, there is always a “but” because the old certainties are giving way to new challenges and opportunities. Patients with Shiga toxin–associated E. coli OH157:H7 HUS may also have low serum C3 concentrations, and some even have mutations in complement regulatory genes.

Noris M, Mescia F, Remuzzi G: STEC-HUS, atypical HUS and TTP are all diseases of complement activation, Nat REV Nephrol 8:622–633, 2012.

176. What is urotherapy?
Urotherapy comprises a range of nonsurgical and nonpharmacologic interventions to improve voiding habits in children with micturition disorders. These include improved voiding postures, regular voiding, and treatment of constipation.
177. Why is constipation associated with daytime urinary symptoms?
Persistent dilatation of the rectum is known to be associated with bladder detrusor overactivity and urgency. Aggressive correction of constipation can improve symptoms in some children with daytime incontinence and overactive bladders.

Loening-Baucke V: Prevalence rates for constipation and faecal and urinary incontinence, Arch Dis Child
92:486–489, 2007.

178. What is vaginal reflux?
Vaginal reflux, also known as vaginal entrapment or vaginal voiding, is characterized by urine leakage after normal voiding in the absence of other lower urinary tract symptoms. It occurs due to trapping of urine in the vagina from poor toilet posture. It is seen in prepubertal girls with wetting of undergarments 10 to 15 minutes following a normal void. Treatment is reassurance and postural modification to ensure complete vaginal emptying.
179. The mother consults Dr. Google when the hematuria recurs and requests an ear, nose, and throat (ENT) consult because she has read that tonsillectomy may cure IgAN. Is this likely to work?
Alas, no. In a 2014 report of a multicenter, randomized, controlled trial, tonsillectomy combined with steroid pulse therapy had no beneficial effect over steroid pulses alone in attenuating hematuria or increasing the incidence of clinical remissions.

Kawamura T, Yoshimura M, Miyazaki Y, et al: A multicenter randomized controlled trial of tonsillectomy combined with steroid pulse therapy in patients with immunoglobulin A nephropathy, Nephrol Dial Transplant 29:1546–1553, 2014. IgA Nephropathy Support Network: www.igansupport.org. Accessed on Mar. 20, 2015.

180. A 15-year-old black girl with sickle cell trait has gross hematuria for the second time 1 day after playing in a lacrosse competition. Could her sickle cell trait
be a factor? Individuals with sickle cell trait may have spontaneous macroscopic hematuria. They are also at greater risk for hematuria and/or rhabdomyolysis after strenuous exercise. In addition, hematuria is the most common presenting sign of renal injury. In athletes it may indicate a benign entity such as exercise-induced hematuria or a more serious injury. Exercise-induced hematuria can originate in the kidney, bladder, urethra, or prostate. The type of activity, as well as activity duration and intensity, contributes to its development. The full differential diagnosis must be considered if hematuria persists
>24 to 72 hours. Renal trauma can occur from a direct blow or deceleration usually in contact and collision sports. Most sports-related renal trauma is mild and can be managed expectantly.
A sporting injury rarely results in nephrectomy. Determining when and if the athlete can resume sport after kidney injury is a difficult issue that requires patient education and an individualized approach.

Holmes FC, Hunt JJ, Sevier TL: Renal injury in sport, Curr Sports Med Rep 2:103–109, 2003.

181. What are the most possible causes of hypertension by age group?
See Table 12-10.

Table 12-10. Causes of Pediatric Hypertension (in Order of Prevalence)
First year of life Secondary (99%) Coarctation of the aorta Renovascular*
Renal parenchymal disease Miscellaneous causes† Neoplasia (4%)
Endocrine (1%)
1-12 years Secondary (70-85%) Renal parenchymal disease Coarctation of the aorta Reflux nephropathy Renovascular
Endocrine Neoplasia Miscellaneous
Primary (essential) (15-30%)
12-18 years Primary (essential) (85-95%) Secondary (5-15%): Same as 1-12 years
*Renal artery/vein thrombosis, renal artery stenosis.
†Bronchopulmonary dysplasia, patent ductus arteriosus, intraventricular hemorrhage. Brady T, Siberry GK, Solomon B: Pediatric hypertension, Contemp Pediatr 25:49, 2008.

182. Why doesn’t eating more protein restore the serum albumin concentration to normal in individuals with the nephrotic syndrome? The loss of urinary albumin is only part of the story. Normally, very small amounts of albumin are filtered at the glomerulus, and filtered albumin is then catabolized by the proximal tubular cells. Amino acids are reabsorbed from the tubular lumen back into the body and resynthesized into albumin in the liver. In patients with the nephrotic syndrome, significantly more albumin is filtered than is normal. Even with increased catabolism and amino acid reabsorption by the renal tubules, there is a limit to how much the liver can make up for urine albumin losses, and therefore the serum albumin concentration decreases. Increasing the protein intake cannot overcome the maximal rate of liver albumin synthesis.
183. For which patients with MCNS should additional therapies be considered?
• No response to the initial course of prednisone
• Nonresponsive to prednisone during relapses
• Frequent relapses
• Steroid dependency
• Severe side-effects from steroids, such as hypertensive encephalopathy, decreasing height velocity, psychosis, morbid obesity, posterior lenticular cataracts
184. What are the alternative therapies for children with MCNS?
Several agents, each with pros and cons, are available.
• Cyclophosphamide often produces a prolonged and sometimes permanent remission.
• Tacrolimus can maintain corticosteroid-free remissions, but relapses often occur when the tacrolimus is stopped.
• Mycophenolate mofetil may be corticosteroid sparing in some patients.
185. What must patients, family members, and prescribers be aware of when offering cyclophosphamide or tacrolimus?
Each may have unacceptable side effects and each must be monitored carefully. Neither is a cure for MCNS. The most important potential side effects of cyclophosphamide are severe varicella infections, alopecia, leukopenia, sterility, hemorrhagic cystitis, and potential malignancy. The most important potential side effects of tacrolimus are hypertension, hyperkalemia, diabetes mellitus, an elevated serum creatinine, renal interstitial fibrosis, and potential malignancy. It is important to stress that with careful monitoring these complications are infrequent.
186. What are the risks associated with the infusion of albumin with furosemide? The administration of 25% albumin and furosemide may have important potential risks. The assumption in this treatment is that the fluid drawn back into the intravascular space by the albumin infusion will be excreted by the kidneys after the administration of furosemide. This may not be true when the nephrotic syndrome is associated with decreased renal function or in patients with pathology other than MCNS and who have a normal or expanded intravascular volume. In such cases, the infusion may lead to intravascular volume overload, hypertension, and pulmonary edema. Pulse rate, respiratory rate, and blood pressure should be monitored frequently during the infusion and the rate slowed or stopped
if signs of fluid overload develop.
187. What physical findings should prompt a search for an underlying renal abnormality?
• Oligohydramnios, neonatal ascites
• High imperforate anus, perineal hypospadias, bladder exstrophy, ambiguous genitalia, prune belly syndrome, caudal regression syndrome, acral cleft, tethered cord, meningomyelocele
• Anuria-oliguria (especially in neonate), abnormal urinary stream, recurrent UTIs, persistent wetness, enuresis in older children
• Wilms tumor with aniridia, hemihypertrophy, abdominal mass
188. A young girl diagnosed with AIN has developed a painful red eye. Could there be a connection?
Yes, there most likely is a connection. It is important to refer to an ophthalmologist for definitive diagnosis and eye therapy. If this is anterior uveitis, there are several possible associations: sarcoidosis, Sj€ogren syndrome, and the syndrome of tubulointerstitial nephritis with uveitis (TINU). In the absence of pulmonary findings, the most likely diagnosis is TINU. In this syndrome, there may be a long delay in diagnosis because the uveitis may not appear for weeks to months after the onset of symptoms.

Signs and symptoms of TINU include persistent or intermittent fever with no etiology, abdominal pain, and unexplained weight loss. The urine findings are minimal with low-grade proteinuria and mild pyuria. The serum creatinine may increase and there may be a partial Fanconi syndrome. Steroids are helpful for the uveitis and the interstitial nephritis. The renal abnormalities and/or eye findings may become chronic.
189. Which patients with UTIs are at higher risk for having an abnormality?
• Boys with a first UTI
• Recurrent UTI or bacteremia
• Infection with an unusual (non–E. coli) organism
• Abnormal renal function or abnormal urinary tract on antenatal screening
• Poor urinary stream
• Prolonged clinical course (symptoms >72 hours) or failure to respond to antibiotic therapy
• Palpable kidneys or abdominal mass
190. How does altering the urine pH affect renal calculi?
• Calcium oxalate stones are the most common type of renal calculi and are unaffected by the urine pH. These stones can be treated with thiazide diuretics, which increase renal calcium reabsorption and thereby decrease the urinary calcium excretion. Potassium citrate can be added if the urinary citrate excretion is low. Calcium intake should not be restricted unless intake is excessively high.
• Calcium phosphate stones may benefit from urinary acidification (in the absence of distal renal tubular acidosis, where lowering the urine pH is not possible).
• Uric acid stones form in acidic urine and also respond to alkalinization to a pH higher than 6.5. Additional therapy includes a reduction in purine intake and occasionally the use of allopurinol to block the formation of uric acid.
• Cystine stones benefit from urine alkalinization (pH >7), high fluid intake, and penicillamine to reduce likelihood of formation.
• STRUVITE or infection stones form in extremely alkaline urine. Urine acidification and antibiotics are the cornerstones of treatment for these stones.
191. In addition to reflux, what other pathologic bladder findings may be noted on a VCUG?
• Diverticula may be seen, especially in the presence of outflow obstruction. A posterior urethral valve or urethral stricture can be detected during the voiding phase of the study. A ureterocele may be seen and appears as a filling defect in the bladder. Other features to note are the bladder capacity, residual volume after voiding, and thickening of the trabeculation of the bladder from bladder hypertrophy.
192. What are the pros and cons of a VCUG versus a radionuclide cystogram (RNC) for the evaluation of reflux?
• Both studies require the catheterization of the bladder and the filling of the bladder with an imaging solution.
• The RNC allows for continuous monitoring of the filling and emptying of the bladder compared with the intermittent fluoroscopy that occurs with the VCUG.
• RNC has about 100 times less the absorbed radiation dose compared with a single radiograph VCUG.
• The VCUG provides greater anatomic detail of the urethra and bladder and can aid in the evaluation of bladder dysfunction better than the RNC.
• The VCUG is more accurate for assessing and grading the degrees (I to V) of reflux.